MOC EXAM Questions for Child Neurology From Practice Test 2

¡Supera tus tareas y exámenes ahora con Quizwiz!

An 11-month-old boy presents to the emergency department with one month of non-bloody, non-bilious vomiting and two weeks of progressively worsening balance. He started sitting independently at 6 months of age but has not been able to do so for the past week. On examination, he is alert but irritable, with impaired upgaze and significant truncal ataxia. Head CT shows a high density, enhancing posterior fossa mass with significant hydrocephalus. Follow-up head MRI reveals a round, slightly lobulated mass within the cerebellar vermis and fourth ventricle, which is isointense to gray matter on T1- and T2-weighted images and is homogenously enhancing. What is the most likely diagnosis? ] Hemangioblastoma Medullary glioma Teratoma Medulloblastoma Juvenile pilocytic astrocytoma

Correct Answer is: Medulloblastoma While many posterior fossa tumors present with similar symptoms, the imaging results suggest a medulloblastoma. Astrocytomas are more common than medulloblastomas but are predominantly cystic. Medullary gliomas, teratomas, and hemangioblastomas are all far less common and would be expected to have different imaging characteristics. While several different imaging modalities are used in the management of patients with medulloblastomas, the diagnosis is still made based on WHO-defined histological criteria. Classic medulloblastoma is characterized by sheet-like areas of small, round, blue cells with scant cytoplasm and dense hyperchromatic nuclei. Homer-Wright rosettes, which are circular groupings of tumor cells, are present in less than half of cases. Other subtypes of medulloblastoma include desmoplastic, large cell anaplastic, medullomyoblastic, and melanotic variants. Staging requires a gadolinium-enhanced MRI of the brain and entire spine and a lumbar puncture to assess for disseminated disease, which is present in 30% of children at presentation; however, drop metastates that are readily apparent on imaging may obviate the need for a lumbar puncture. CSF should be obtained 2 weeks post-operatively to avoid false positive result

A 12-year-old girl presents to the emergency department with a 36-hour history of fever (103o F.), headache, and altered consciousness. On examination, she does not have any meningeal signs. She requires repeated tactile stimulation to maintain arousal, is only oriented to person, makes frequent paraphasic errors, and can recall just one of three objects after a distracting task. The remainder of her neurologic examination is unremarkable. CSF analysis reveals a red blood cell count of 23 cells/hpf; white blood cell count of 114 cells/hpf with a lymphocytic predominance; glucose of 73 mg/dL; and protein of 97 mg/dL. Given this information, which treatment should be initiated at this time? Iduroxidine Ceftriaxone Vidarabine Solumedrol Acyclovir

Correct Answer is: Acyclovir This patient presents with features typical of herpes simplex encephalitis (HSE), which affects 1 in 250,000 to 1 in 500,000 individuals per year in the United States, with approximately one-third of cases occurring in patients between the ages of 6 months and 20 years. HSE can occur in the setting of a primary infection or a reactivation of the herpes simplex virus (HSV). Patients present with a variety of non-specific findings, including fever, headache, altered consciousness, dysphasia, seizures, and/or focal neurologic deficits. Approximately 5-10% of patients have normal CSF studies on their initial assessment. More often, CSF examination reveals an elevated white blood cell count with a lymphocytic predominance and an elevated protein. Red blood cells are often present but are non-diagnostic. The diagnosis is confirmed by PCR detection of HSV DNA in the CSF—a test which has a sensitivity of 94% and specificity of 98%. Other supportive studies include EEG, CT, and MRI. Treatment consists of intravenous acyclovir 10 mg/kg every 8 hours (30 mg/kg/d) for 14-21 days. Idoxuridine was the first antiviral drug used to treat HSE; however, it proved to be both ineffective and toxic. Acyclovir was demonstrated to be superior to vidarabine in a head-to-head trial. Neither ceftriaxone, nor solumedrol would be appropriate treatments, as both the clinical presentation and CSF studies suggest viral encephalitis

A 1 month old infant has been colicky since he was discharged from the hospital nursery. He eats well, is growing, fixes and follows his mother's face, startles normally to loud noise. But he has episodes of crying vigorously during which he pulls his legs up and appears to be uncomfortable. His parents cannot console him during these episodes, but eventually, he falls asleep and wakes up fine. His pediatrician tells them he will outgrow these episodes. One evening, his parents decide to go out to a restaurant for dinner. They are grateful to the 17 year old boy who lives next door, as he has agreed to stay with the baby to give them a well-deserved respite. When they come home, they pay the young man and thank him. As soon as he leaves, they go into the baby's room and notice the baby looks pale and is breathing funny. They turn on the light and try to arouse the baby, but he does not wake up. The mother screams as the father calls 9-1-1. He is rushed by ambulance to the hospital. A quick exam reveals a comatose infant with stable vital signs and retinal hemorrhages. His skin, chest, abdomen, and limbs appear normal. A CT scan of his head shows right occipital, left temporal, and bifrontal intraparenchymal hemorrhages and a moderate amount of subarachnoid blood. You consult your appropriate colleagues and order a: Stat MRI scan of his head Spine MRI Lumbar puncture Serum S100b Skeletal survey

Correct Answer is: Skeletal survey Shaken baby syndrome would appear to be the diagnosis in this infant. The retinal hemorrhages, subarachnoid blood, and locations of intraparenchymal blood all point to traumatic brain injury, and the social story is suspicious for but never diagnostic of non-accidental trauma. One cannot assume that shaking, if it occurred, was the only traumatic event or that the circumstances that led to the baby's current state constituted an isolated incident. For this reason, a skeletal survey is typically performed in children in whom non-accidental trauma is suspected

Children with Crouzon syndrome can develop cognitive impairment. This is because of: Cerebral dysgenesis Hypoxia from airway dysgenesis Abnormal calcium metabolism Mitochondrial dysfunction Multiple craniosynostoses

Correct Answer is: Multiple craniosynostoses Management of the multiple simultaneous synostoses of the skull in Crouzon syndrome is an urgent matter, as they can restrict outward brain growth in all directions and limit CSF egress through occlusion of the foramina by impingement from inward-expanding parenchyma.

A 7-year-old boy is admitted to the ICU after an out-of-hospital cardiac arrest. He was being treated for vomiting and headache that became daily after initially being intermittent, paroxysmal and focal on the right side. There was a family history of migraine and he reported visual blurring at the onset of vomiting and headache in the left visual field. He had a non-contrast CT scan that was unremarkable and no sign of intercurrent illness. He was initially treated with sumatriptan but as this produced no real relief he was hospitalized and given ondansetron with analgesia. After two days in the hospital he was discharged slightly improved on scheduled ondansetron and acetaminophen. The following morning he was discovered pulseless in his bed. His father began CPR and emergency medical technicians arrived within 5 minutes. He was intubated and ECG showed ventricular fibrillation. By the time he was transported to the ER at 30 minutes he had a sinus rhythm and blood pressure. His exam in the ER showed no pupillary light reflex and absent corneal reflex, extraocular movements, and gag. CT scan was normal except for indistinct gray-white differentiation. An EEG at 10 hours post-arrest showed electrocerebral silience. Evaluation for cause of his arrest was negative except for the presence of a prolonged QT interval of 0.48 suggesting he may have acquired long QT syndrome secondary to ondansetron. A repeat EEG shows electrocerebral silence and his exam is unchanged. What is your approach with the family? "If only I had seen this child..." You explain that you never use ondansetron for nausea and vomiting in the setting of migraine and your approach to migraine would have likely resulted in resolution of his symptoms prior to discharge making only rescue medications necessary. You suggest they withdraw support since he is brain dead on your exam. "Your child is dead according to my tests..." You explain that after cardiac arrest loss of pupillary light response is associated with death or vegetative state 95% of the time and he has not improved so he cannot recover. The EEG confirms your opinion. "Let me summarize what I know about your son..." You describe the history as you know it and then your exam and test results describing how you arrive at the conclusion the child is brain dead. After asking for questions you give your opinion that support should be withdrawn. "To be sure we understand each other, could you tell me what you understand is going on with your son..." You listen to the family's ideas and answer any questions that arise from your opening statement and then explain your exam and the EEG results. After allowing questions and discussions with the family you state your recommendation that support be withdrawn. "The studies and my exam show there is no hope..." You present the data from the CT scan, EEG and your exam and state that the child is brain dead. You wait for questions and when there are none excuse yourself.

Correct Answer is: "To be sure we understand each other, could you tell me what you understand is going on with your son..." You listen to the family's ideas and answer any questions that arise from your opening statement and then explain your exam and the EEG results. After allowing questions and discussions with the family you state your recommendation that support be withdrawn. The approach in "d" addresses many of the issues that have been identified in the literature on end-of-life care. In the case of children the parents' views of their and their child's rights to act autonomously is the foremost principle of contemporary medical ethics, but the importance of autonomous choice to individual parents varies greatly, with some preferring to delegate or share decision making with family members or physicians. Physicians need to determine each family's preferences in this regard. This is one of many areas where communication with the family is important. The list shows the areas identified in reports (see reference) that are crucial to good ICU communication. Identify a private place for communication with family members Increase proportion of time spent listening to family rather than talking Identify commonly missed opportunities Listen and respond to family members Acknowledge and address family emotions Explore and focus on patient values and treatment preferences Explain the principle of surrogate decision making to the family Affirm nonabandonment of patient and family Assure family that the patient will not suffer Provide explicit support for decisions made by the family Use of the VALUE mnemonic during family conferences: V = value statements made by family members A = acknowledge emotions L = listen to family members U = understand who the patient is as a person E = elicit questions from family members Provide consistent communication from different team members

A 4 year old girl undergoes a tonsillectomy. She awakens from the anesthesia in the recovery room and vomits twice. The ENT surgeon gives the recovery room nurse a verbal order for metaclopromide. Thirty minutes later, the girl begins having frequent episodes in which her mouth and eyes are pulled to the side and her neck is twisted to the same side. She seems quite distressed and is mute during these episodes, each of which lasts 30 seconds to a minute. She should be given: An EEG Another dose of metaclopromide A neck brace A dose of diphenhydramine A blanket from home

Correct Answer is: A dose of diphenhydramine Oculogyric crisis is a dystonic movement disorder most often induced by drugs, and commonly seen with phenothiazines. Patients are awake and alert and quite distressed by the "pulling" up of their eyes and contortions of their facial musculature. At its worst, this disorder can interfere with breathing. Diphenhydramine (IV) generally gives rapid relief, but must be continued orally or IV thereafter, often for more than 24 hours, as the half-lives of phenothiazines are relatively long

A 6-year-old boy has been having fits of screaming at night for the past 2 months, sometimes several times nightly. Typically, about an hour after falling asleep he sits up in bed, opens his eyes and begins screaming, is tremulous, sweaty and appears frightened. After a few minutes, the episode stops and he goes back to sleep. Last night he got out of bed and ran around the house while screaming. His parents cannot console him during the fits and he has no recollection of them the following morning. He snores loudly each night and sometimes gasps for air. His past medical history is remarkable for recurrent episodes of otitis media when younger, for which he had myringotomy tubes placed. His early development was normal, but he is hyperactive and inattentive in first grade. On examination, he is a mouth breather and has enlarged tonsils. Which of the following is the best treatment option? Parental reassurance Adenotonsillectomy Benzodiazepine Oxygen Selective serotonin reuptake inhibitor

Correct Answer is: Adenotonsillectomy This boy is having typical sleep terrors (aka night terrors, pavor nocturnus), which are a form of non-REM parasomnias arising from slow wave sleep. They affect about 3% of children, with peak incidence at 5-7 years of age, usually resolving by adolescence. They are characterized by marked autonomic nervous system activation: tachycardia, tachypnea, tremulousness, mydriasis, and sweating. Facial expression of intense fear is associated with uncontrollable shouting, screaming, gasping, moaning, and agitation. Most children remain in bed during the events, but sometimes will walk or run during them. They typically last one to several minutes, sometimes up to half an hour. During them, the child cannot be consoled and attempts to awaken the child from them make them more intense. Differential diagnosis includes nightmares (briefer, latter half of the night, consolable, dream recollection), seizures (1-2 minute stereotyped hypermotor activity), panic attacks, and cluster headaches. They are enhanced by acute stress, sleep deprivation, and some medications (stimulants, sedative-hypnotics, neuroleptics, antihistamines). Studies have shown that sleep disordered breathing and restless legs syndromeperiodic limb movement disorder can augment sleep terrors; treatment of these conditions can significantly improve the terrors. In the patient presented, signs and symptoms of obstructive sleep apnea are present and should be addressed.

A 6 year old child is brought to you by her adoptive parents. She was adopted from a Rumanian orphanage. She has telangiectasias on her face and sclerae and walks with a walker because she is ataxic. When the adoptive parents ask about the long-term implications of her genetic disease you should tell them she is at higher risk than unaffected children of developing: Tuberculosis Inadequate responses to vaccinations Sinus infection Cancer All of the above

Correct Answer is: All of the above Ataxia-telangiectasia is a rare inherited disorder that affects the nervous system, immune system, and other body systems. This disorder is characterized by progressive ataxia beginning in early childhood. Telangiectasias, which occur in the eyes and on the surface of the skin, are also characteristic of this condition. Mutations in the ATM gene, a gene the product of which is involved in DNA repair, cause ataxia-telangiectasia. Patients with ataxia-telangiectasia have immunoglobulin deficiencies and many develop chronic lung and sinus infections, including tuberculosis. They also have an increased risk of developing cancer, particularly leukemia and lymphoma. Affected individuals are very sensitive to the effects of radiation exposure. The life expectancy of people with ataxia-telangiectasia varies greatly, but affected individuals typically live into early adulthood.

A 6-month-old child has developed recurrent events consisting of flaccid paralysis of one side of the body, or the other, with complete recovery of motor function between individual episodes. There is no prior history of medical problems or developmental concerns, and the family history is negative for similar symptoms. The individual events last for approximately 4 hours, and do not seem to be distressing for the child. The mother reports that some of the events seem to be precipitated by a bath or sudden change in environmental temperature. Consciousness during the spell appears normal. Brain MR imaging and MR angiography are normal. VideoEEG seizure monitoring captured one event, showing EEG slowing over the cerebral hemisphere contralateral to the weakness, but no epileptiform abnormalities. What is the single most likely diagnosis for this patient? Multiple sclerosis, relapsing and remitting type Temporal lobe epilepsy Transient ischemic attack Alternating hemiplegia of childhood Segawa disease

Correct Answer is: Alternating hemiplegia of childhood Alternating hemiplegia of childhood is rare, occurring in perhaps 1 in 1,000,000 births. The majority of patients present with first symptoms during infancy or even in the newborn period. The pathogenesis of unknown, although some clinical features are shared with conditions such as migraine and mitochondrial disorders (mitochondrial encephalopathy with lactic acidosis and stroke-like episodes [MELAS]). Large vessel occlusion or vasospasm appears to be an unlikely mechanism.

A 10 year old boy woke up this morning noticing that he could see better out of his right eye than out of his left eye. His vision was blurry in the left eye and rubbing the eye or squinting did not make it better. He had no pain in the eye and had not had prior trauma, fever, respiratory symptoms, or headache. He told his mother and she made an appointment for him to be seen later that day by her optometrist. However, a few hours later, before this appointment occurred, the boy experienced mid-back pain and rapidly developed difficulty urinating, stiffness of his legs, and tingly sensation when he ran his hand over his abdomen below his belly button. His mother brought him to a local emergency room. He was admitted for evaluation and treatment. The physicians most likely checked him for: Anti-NMDA receptor antibodies Anti-microsomal antibodies Anti-thyroid antibodies Anti-aquaporin 4 antibodies Anti-neuronal antibodies

Correct Answer is: Anti-aquaporin 4 antibodies Neuromyelitis optica (NMO) is an uncommon disease syndrome of the central nervous system (CNS) that affects the optic nerves and spinal cord. Individuals with NMO develop optic neuritis and transverse myelitis. NMO leads to loss of myelin. Historically, NMO was diagnosed in patients who experienced a rapid onset of blindness in one or both eyes, followed within days or weeks by varying degrees of paralysis in the arms and legs. In most cases, however, the interval between optic neuritis and transverse myelitis is significantly longer, sometimes as long as several years. After the initial attack, NMO follows an unpredictable course. Most individuals with the syndrome experience clusters of attacks months or years apart, followed by partial recovery during periods of remission. This relapsing form of NMO primarily affects women. The female to male ratio is greater than 4:1. Another form of NMO, in which an individual only has a single, severe attack extending over a month or two, is most likely a distinct disease that affects men and women with equal frequency. The onset of NMO varies from childhood to adulthood, with two peaks, one in childhood and the other in adults in their 40s. In the past, NMO was considered to be a severe variant of multiple sclerosis (MS) because both can cause attacks of optic neuritis and myelitis. Recent discoveries, however, suggest it is a separate disease. NMO is different from MS in the severity of its attacks and its tendency to solely strike the optic nerves and spinal cord at the beginning of the disease. Symptoms outside of the optic nerves and spinal cord are rare, although certain symptoms, including uncontrollable vomiting and hiccups, are now recognized as relatively specific symptoms of NMO that are due to brainstem involvement. The recent discovery of the anti-aquaporin 4 antibody in many individuals with NMO gives doctors a reliable biomarker to distinguish NMO from MS. The antibody, known as NMO-IgG, seems to be present in about 70 percent of those with NMO and is not found in people with MS or other similar condition

A 12-year-old girl presents with a 2 day history of weakness in the lower extremities and difficulty with bladder control. One year ago she had an episode of loss of vision in her left eye which gradually resolved over a period of several weeks. On examination she has a mild afferent pupillary defect and 20/80 vision in her left eye. She has increased tendon reflexes in the lower extremities and extensor plantar responses. No sensory level is appreciated. Which of the following antigens might be useful in establishing the diagnosis in this patient? Alpha-4-beta integrin (ά4β1) Acetylcholine receptor (AChR) Voltage gated calcium channel (VGCC) Aquaporin 4 channel (AQP4) Anti Muscle specific kinase antibody (MuSK)

Correct Answer is: Aquaporin 4 channel (AQP4) The integrin protein is important in cell signaling in glial neuronal interactions. The AChR is the primary ion channel in the neuromuscular junction and is important in impulse transmission as is the calcium channel. The MuSK antibody is associated with a form of Myasthenia. Although the role of the aquaporin channel is incompletely understood, it is associated with neuromyelitis optica and distinct from multiple sclerosis

A local pediatrician calls you for advice on a patient he has been seeing. The patient is a 12 year old boy who has not been to school for 3 months because of headache. He has been getting "homebound education" and has not been doing well; the school wants him to repeat the year. His headaches are there all day every day and are not pulsatile. They move from place to place on his head and are mildest upon awakening and most severe by the end of each day. His past medical history is remarkable for mild asthma for which he takes a prn inhaler. His family history is remarkable for depression in his mother and sleepwalking in his younger brother. His maternal grandmother had menstrual headaches that required her to sleep when they occurred. The parents are divorced and there is a restraining order against the father because he physically abused the mother. The child's general and neurological examinations have been normal on multiple occasions. The pediatrician is frustrated because the child has had no relief with nightly amitriptyline plus sumatriptan twice a week for the worst of the headaches. You should best advise the pediatrician to: Obtain a CT scan of the child's head. Switch from amitriptyline to nortriptylline. Tell the mother to have the restraining order against the father lifted. Get toxic screens on the child's blood and urine in case the father poisoned him. Arrange for psychiatric evaluation and counseling for the mother and children

Correct Answer is: Arrange for psychiatric evaluation and counseling for the mother and children. This youngster has chronic daily headache with many psychosocial stressors likely contributing to it. He does have a family history of migraine and phenomena that frequently cosegregate with migraine in families (i.e., sleepwalking), but his headaches do not have the character or temporal features of common migraine. His school absence necessitates urgent action and return to school may be therapeutic for this young man but will likely require considerable psychological and scholastic support

A 7-year-old girl presents to the urgent care after she was seen wandering around the playground, and then vomited at school. The teacher who is present with her at the urgent care described her as clumsy, which was uncharacteristic for her. On examination, she is pale and complains of nausea. She is also complaining of a headache in the back of her head, which the teacher says started after arrival in the urgent care. When the mother arrives, she states that the child did this in a similar fashion twice before, over the past 4 months. Mother describes that the child is generally is back to baseline after an extended nap. Which of the following is the most likely diagnosis? Abdominal migraine Acute confusional migraine Acute post-traumatic migraine, after unwitnessed fall on the playground Basilar migraine Cyclic vomiting syndrome

Correct Answer is: Basilar migraine Basilar migraine is one of the complicated migraine variants, likely the most common, said to represent 3-19% of all migraine. The mean age of onset is 7 years. The diagnosis usually requires the presence of 2 or more clear signs and symptoms of posterior fossa involvement. These include: vertigo, nausea or vomiting, ataxia, visual field deficits, diplopia, tinnitus, hearing loss, confusion, dysarthria, weakness, syncope, with the first 4 being the most common. The typical child will begin with the posterior fossa symptoms such as intense dizziness, vertigo, ataxia and diplopia, which can last for minutes to an hour, and then are followed by the headache phase. The headache is often occipital in location.

A 16-year-old girl whose parents are friends of yours is evaluated for possible seizure activity. She was observed to have episodes of staring, morning jerks, and had one convulsion. A diagnosis of juvenile myoclonic epilepsy (JME) was made and confirmed on EEG. Additional testing revealed that she tested positive for marijuana and cocaine. She was also found to have genital herpes. The patient asks that her parents not be told of her drug or sexual activities. Which of the following is the most appropriate management? Begin treatment for JME with valproate and inform her parents of the social problems described. Begin treatment for JME with valproate and consult your state law on disclosing information to parents. Begin treatment for JME with lamotrigine and inform her parents of the social problems described. Begin treatment for JME with lamotrigine and consult your state law on disclosing information to parents. Refrain from treatment of her JME until social problems described are adequately resolved.

Correct Answer is: Begin treatment for JME with lamotrigine and consult your state law on disclosing information to parents. The patient presents with a relatively obvious epileptic syndrome. Given her sexual proclivities, the use of valproate should be avoided because of its potential teratogenic effect. The main issue has to do with what to tell the parents. Child neurologists might disagree as to whether we think the patient's welfare is best served by discussing the problems with the parents or abiding by the secrecy requested by the patient. However, we need to be aware laws vary from state to state and need to be carefully addressed before decisions are applied.

A 12-year-old boy has a recent history for paroxysmal events shortly after transitioning to sleep at night. He is otherwise normal, without significant academic or behavioral problems. Brain MRI was normal, but video-EEG seizure monitoring confirmed the presence of a subtle but consistent ictal pattern over the frontal head regions at the time of the spells. The diagnosis is supported by a history of nocturnal seizures in an older brother and maternal uncle. Which of the following is the most accurate description of the seizures expected in this patient? Unresponsive staring, with chewing movements of the mouth, and non-purposeful fumbling movements of the hands. Sudden rage behaviors, with directed violence, after minor frustration Sudden, shock-like contractions of head, neck, and both arms Brief paroxysmal events during sleep with thrashing and yelling Throbbing headaches associated with vomiting

Correct Answer is: Brief paroxysmal events during sleep with thrashing and yelling Autosomal Dominant Nocturnal Frontal Lobe Epilepsy (ADNFLE) has been associated with gain-of-function mutations in genes encoding subunits of the neuronal nicotinic acetylcholine receptors (nAChRs) in some affected families. The seizures are frequent, brief, usually nocturnal, and may involve unusual motor features with posturing or thrashing with intact awareness. The ictal change on the EEG may be difficult to define due to muscle and movement artifact, and onset of seizure activity from mesial frontal or orbitofrontal regions. Most patients respond well to conventional antiepilepsy drug (AED) therapy.

A 16-month-old girl presents to the emergency department with a 6-day history of "twitching" that started in her legs but eventually affected all four extremities. Since the onset of her symptoms, she has gradually lost the ability to ambulate. One day prior to presentation, her mother noted that her eyes were moving abnormally. She has not had any recent viral infections or immunizations. Neurologic examination is significant for involuntary, chaotic, multi-directional saccadic eye movements, action myoclonus, and both appendicular and truncal ataxia. Head MRI is normal and CSF analysis shows a mild pleocytosis and a mildly elevated protein. Urine catecholamines are notable for a homovanillic acid of 61.5 mg/g (normal 0-32.6) and vanillyl mandelic acid of 20 mg/g (normal 0-16.7). Which of the following is the most appropriate next step in management? CT scan of the chest and abdomen Anti-Hu antibody titers Ceruloplasmin level CSF measles antibody titers ENT consultation

Correct Answer is: CT scan of the chest and abdomen This patient presents with opsoclonus-myoclonus-ataxia (OMA) syndrome, also referred to as "dancing eye and dancing feet syndrome." The term opsoclonus refers to involuntary, chaotic, multidirectional saccades, which are present during fixation and smooth pursuit and persist during eyelid closure and sleep. Opsoclonus often occurs in junction with appendicular myoclonus and cerebellar ataxia. Patients may exhibit behavioral disturbances, postural tremor, and/or encephalopathy. Opsoclonus can occur as a paraneoplastic, parainfectious, toxicmetabolic, or idiopathic phenomenon. In children, more than half of paraneoplastic opsoclonus is secondary to neuroblastoma. Although a variety of antineuronal antibodies have been identified in this disorder, the majority of patients are seronegative. All children presenting with OMA should be evaluated for occult neuroblastoma with urine catecholamines, including homovanillic acid and vanillyl mandelic acid, imaging of the chest and abdomen, and, if negative, 123I-metaiodobenzylguanidine (MIBG) scan. When negative, the evaluation should be repeated after several months. The prognosis for survival of neuroblastoma patients with opsoclonus is better than for those without opsoclonus. Management involves treatment of the underlying process, as well as symptomatic treatment with corticosteroids, intravenous immunoglobulin (IVIG), or adrenocorticotrophic hormone (ACTH). Low-dose cyclophosphamide may be beneficial in refractory cases. With or without treatment, children with opsoclonus and neuroblastoma usually experience resolution of the opsoclonus, though most experience relapses

An 8 month-old girl is being seen for evaluation of a seizure described as staring followed by a generalized convulsion lasting 5 minutes. She has a history of hypotonia and developmental delay, is unable to roll over and does not reach for objects with her hands. On examination, her head circumference is above 95th percentile, while her weight and length are at 25th percentile. She has optic atrophy and nystagmus, central hypotonia and increased tone in the ankles, with clonus. Her brain MRI shows increased signal in the white matter diffusely on T2-weighted images. Her MRS shows a strikingly elevated NAA peak. Which of the following is the most likely diagnosis? Krabbe leukodystrophy Adrenoleukodystrophy Mitochondrial encephalomyopathy Canavan disease Metachromatic leukodystrophy

Correct Answer is: Canavan disease The triad of hypotonia, macrocephaly, and head lag in an infant after the age of three to five months should raise the suspicion of Canavan disease. Diagnosis of Canavan disease in symptomatic individuals relies upon demonstration of very high concentration of N-acetyl aspartic acid (NAA) in the urine. NAA concentration is also elevated in the blood and cerebrospinal fluid (CSF) of children with Canavan disease. In the brain, the elevated NAA can be visualized by 1H MRS as a single peak with a resonance of 2.01 ppm on the spectrum

A 10-year-old girl has recently been admitted by the pediatric gastroenterologists for vomiting and anorexia, and was diagnosed with acute hepatitis secondary to non-A, non-B hepatitis. She has not responded well to supportive care, and has developed a significant coagulopathy, requiring treatment with cryoprecipitate and recombinant Factor VII (rFVII). She has become significantly obtunded over the past 24 hours, and a pediatric neurology consultation is requested. On examination, she will open her eyes to voice, but cries in response to questions, and will not respond verbally. Her parents describe her as confused. Her neck is supple and her optic disc margins are blurred. There is no tremor or asterixis. Which of the following is the most likely neurological complication from her systemic disease? Obstructive hydrocephalus Large-vessel ischemic stroke Cerebral edema with compromised cerebral perfusion Coma secondary to elevated serum carnitine levels Hyperglycemia

Correct Answer is: Cerebral edema with compromised cerebral perfusion Acute hepatic failure in children is uncommon. In infants, it is more likely to be associated with an inborn error of metabolism (such as tyrosinemia), whereas in older children and adolescents it is more often due to viral hepatitis or drug exposure (such as acetaminophen). Wilson's disease should also be considered in older children. Children with encephalopathy associated with acute hepatic failure may not have asterixis or tremor. The presence of severe coagulopathy or coma is associated with an increased risk of death. Liver transplantation is required for those children who do not recover spontaneously with supportive care.

A full-term infant was born to a healthy mother after a routine pregnancy, labor and delivery. The baby was noted to be irritable and microcephalic shortly after birth. Cranial ultrasonography and subsequent CT scan showed enlarged posterior horns of the lateral ventricles and punctuate calcifications which are both periventricular and diffuse. Neurological examination demonstrates increased tone and hyperreflexia. A dilated funduscopic exam shows a large area of absent retina with hyperpigmented margins. Which of the following is the most likely diagnosis? Congenital Rubeola Congenital toxoplasmosis Congenital herpes simplex type 1 Congenital rubella Congenital syphilis

Correct Answer is: Congenital toxoplasmosis Congenital toxoplasmosis (CT) is a rare condition despite the ubiquitous presence of the parasite in the environment and high incidence of seropositivity in the population. Domestic cats are the reservoir and human transmission occurs from oral ingestion of ova in fecal contaminated dirt or litter boxes. Congenital infection is thought to occur in 50-80% of infants of mothers who initially acquire the infection during pregnancy, with hemaotgenous spread of the parasite to the placenta and fetus. Although only 5% of children with CT have neurological sequelae approximately 35% develop chorioretinitis by age 12 with many lesions being subclinical. In the US, Massachusetts routinely screens newborns for the condition with a yearly incidence of new cases of 0.8 per 10,000. Treatment is not known to prevent chorioretinitis or neurologic injury but this is debated. The case presented represents the minority of CT cases with only 1 of 22 cases reported in a 2 year period in the UK having this constellation of signs and symptoms. In contrast to congenital CMV where the calcifications involve the periventricular region, diffuse calcification involving the cortex is the rule in CT and may also be periventricular with hydrocephalus. Maternal prenatal screening has been suggested and is carried out in France for example, however, opponents point out that it is expensive, raises anxiety in positives and there is no evidence that maternal treatment prevents CT. The prognosis for a child affected as above is poor.

A 7 year old girl has multiple stereotyped episodes each week. She says that, with each of these episodes, "I see a funny shape in front of my eyes. Then the room feels like it tilts to the side for a little while. Then I get sick to my stomach and feel dizzy. Sometimes it only lasts a few minutes. But if it lasts longer somewhere when I can lie down and go to sleep, I can make it go away." The girl had benign infantile episodic head tilt from 2 months to 6 months of age. Her mother, sister, and maternal grandmother get migraines. The girl has a normal neurological examination, including response to Hallpike maneuver. During a particularly long episode, she had a 30-min "awake only" EEG, and it was normal. Due to the frequency of episodes: Give her a dose of sumatriptan when her next episode starts. Consideration of a preventative medication such as nortrypylline Send her to an ENT surgeon to get tubes put into her ears. Get an MRI scan with particular attention to her posterior fossa. Send her to a physical therapist for vestibular conditioning.

Correct Answer is: Consideration of a preventative medication such as nortrypylline The episodes this girl is experiencing have the time course and accompanying and attenuating features of migraine equivalents. She can make them go away with sleep. The sensory distortions are accompanied by nausea. She has a past medical history of benign intermittent infantile torticollis. She has a positive family history for migraine and a negative awake EEG. She is getting episodes frequently, so prophylactic treatment is advised

A 6-year-old girl is admitted to the Emergency Department after she is struck by an automobile while running across the street. At the scene, the paramedics found her to be awake, but distraught and complaining of neck pain. She was transported with her head and neck immobilized in neutral position. Upon examination, she is breathing well, has stable vital signs, and appears to be moving all four extremities equally. Radiographs of the cervical spine show abnormal widening of the interspace at C1-C2. Which of the following is the most appropriate next step in management? Emergency tracheostomy Continued immobilization of the head and neck, careful observation, and neurosurgical consultation. Flexion and extension views of the neck under fluoroscopy Lumbar puncture to exclude subarachnoid hemorrhage Ophthalmology consultation for dilated fundus examination

Correct Answer is: Continued immobilization of the head and neck, careful observation, and neurosurgical consultation. Cervical spine (C-spine) injuries in pre-pubertal children are relatively uncommon in comparison to older age groups. In young children, C-spine injuries are most likely to occur at the craniocervical junction or upper vertebral segments C1-C4. Children may have severe disruption of ligaments or other soft supporting tissues without obvious abnormalities of the vertebrae on radiographs. However, selected MR sequences are sensitive to such soft tissue injuries, if indicated. Children who are unresponsive, intoxicated, have abnormal findings on neurological examination, or neck pain should remain immobilized until C-spine stability has been determined by the appropriate studies. The role of corticosteroid administration for neuroprotection following spinal injury remains contentious.

A 3 year old boy is seen by his pediatrician for a well-child check-up. The pediatrician thinks he can feel the boy's liver 1 finger breadth below the right costal margin. He gets liver function studies done at a local lab and is called by the lab director with the following abnormal values: ALT,70 U/L; AST,100 IU/L. The lab director also said that his venipuncture technician told him that the child's mother was going to call the pediatrician during callin hours tomorrow because she forgot to tell him the child seems to her to have trouble standing up when he has been sitting on the floor playing with toys. If the lab has additional blood left over, the pediatrician should ask the lab director to test it for: Glucose-6-phosphate Bilirubin Alkaline phosphatase Creatine phosphokinase Prothrombin time

Correct Answer is: Creatine phosphokinase So-called "liver function tests" are tests for enzymes that are expressed at high levels in the liver, but are expressed in many other cell types, including muscle. As a consequence, elevated ALT and AST blood levels occur in neuromuscular disorders that involve lysis or high turnover rates of myocytes. Destruction of muscle in Duchenne muscular dystrophy is one such situation. It can be differentiated from the elevated ALT and AST of liver disease by the concomitant and disproportionate elevation of "muscle function tests", such as levels of CPK and/or aldolase.

A 2-year-old boy presents to the local emergency department with multiple seizures described as either whole body jerks or rhythmic shaking of the extremities, often involving only one side. There were no perinatal difficulties, but he did have a history of recurrent febrile seizures beginning at 9 months of age, 2 of which were prolonged and required high-dose pentobarbital treatment. Early developmental milestones were met on time but the family reports that recently his language skills have lagged behind compared to other children the same age. A routine outpatient EEG demonstrates abundant generalized spike-waves and polyspike waves with diffuse background slowing. A brain MRI scan is normal. Which of the following diagnoses is most likely given this scenario? Benign myoclonic epilepsy of infancy Dravet syndrome Lennox-Gastaut syndrome Doose syndrome Generalized epilepsy with febrile seizures plus

Correct Answer is: Dravet syndrome This patient exhibits many of the features of Dravet syndrome, otherwise known as severe myoclonic epilepsy of infancy. Development is normal prior to the onset of seizures. This condition usually manifests during the first year of life with febrile convulsions, often with unilateral features. Other seizure types that appear later include generalized clonic, tonic-clonic, absence, and myoclonic seizures, or unilateral hemiclonic and/or complex partial seizures. The occurrence of status epilepticus is frequent. Psychomotor retardation is usually observed during the second year after the onset of seizures. Progressive neurologic deficits such as ataxia and corticospinal tract signs develop subsequently. Although the diagnosis of Dravet syndrome is suggested based on electroclinical features, the majority of affected patients have demonstrable mutations in the SCN1A gene which encodes the alpha subunit of a voltage-gated sodium channel.

A 12-year-old girl presents to your office with a 1 month history of progressively worsening involuntary movements that interfere with her ability to write and walk. One week prior to presentation, she began slurring her speech. She has continued to do well in school but has been crying more easily than usual. Her birth and past medical histories are unremarkable, and she has not been taking any medications. Her general examination is normal. On neurologic examination, she is severely dysarthric and has motor impersistence during tongue protrusion. She has mild, diffuse hypotonia and marked generalized chorea, which causes her to bump into things while ambulating. Her deep tendon reflexes are 2+ but are hung-up at the knees. Her primary care physician performed routine blood work, including a basic metabolic panel, complete blood count, sedimentation rate, and pregnancy test, all of which were normal/negative. An ASO titer is positive. An abnormal/positive result on which of the following tests would alter the patient's treatment? C-reactive protein Anti-DNase B Echocardiogram Toxicology screen ESR

Correct Answer is: Echocardiogram Sydenham's chorea (SC) is seen in 10% to 20% of patients with acute rheumatic fever (ARF), usually weeks to months after a group A β-hemolytic streptococcal (GABHS) infection. Clinical hallmarks of the disorder include chorea, hypotonia, and emotional lability. Although the mechanism of disease is incompletely understood, the prevailing theory is that molecular mimicry between streptococcal and basal ganglia peptides leads to immunemodulated damage of the basal ganglia. SC is diagnosed clinically, though elevated antistreptolysin O and/or anti-DNase B titers may corroborate the diagnosis. Negative titers do not exclude the diagnosis. The diagnostic work-up should include a throat culture to determine whether the patient has an active GABHS infection; if positive, the patient should be treated with antibiotics. Additional laboratory and imaging studies may be necessary to exclude other causes of chorea, such as drug reactions, systemic lupus erythematosus, and Wilson's disease. All patients should have an echocardiogram to assess for carditis and should be started on long-term antibiotic prophylaxis to prevent rheumatic recurrences. For patients with chorea that is impairing, treatment options include valproate, carbamazepine, and/or dopamine receptor antagonists.

A 6-year-old girl presents to the office for evaluation of language difficulties. She was born full-term by normal spontaneous vaginal delivery following an uncomplicated pregnancy. She walked at 9 months, said single words at 11 months, and combined words at 20 months. At age 5, her speech became progressively more telegraphic, and she began indicating her needs by pointing or by using sign language. On neurologic examination, she is able to follow simple commands. She has word-finding difficulties in spontaneous speech and on naming tasks and is unable to repeat short phrases. The remainder of her neurologic examination is normal. Formal audiometric testing is unremarkable. Her parents state that she occasionally stares off into space for 30 seconds to 1 minute. She has never had a convulsive seizure, nor does she have a family history of epilepsy. An EEG is most likely to show which of the following patterns? Bilateral periodic epileptiform discharges (BiPEDs) Continuous, diffuse, polymorphic theta and delta slowing Electrical status epilepticus in slow wave sleep Hypsarrhythmia with periods of electrodecrement Normal waking and sleep

Correct Answer is: Electrical status epilepticus in slow wave sleep In 1957, William L. Landau and Frank R. Kleffner reported a "syndrome of acquired aphasia with convulsive disorder in children" and postulated that persistent convulsive discharges were causing a functional ablation of brain areas concerned with language. Since that time, it has become apparent that there are several epileptic encephalopathy syndromes of childhood that are characterized by deterioration of one or more cognitive functions with or without motor, behavioral, and/or psychomotor decline in association with an EEG that shows strong activation of epileptic activity during sleep. At this point, electrical status epilepticus of slow wave sleep (ESES) and continuous spikes and waves during slow wave sleep (CSWS) are considered synonymous terms, and Landau-Kleffner syndrome (LKS) a distinct presentation of ESES/CSWS in which acquired aphasia is the core symptom. LKS typically presents with language disturbance in normally developing children aged 3 and 8 years. Some have argued that the language disturbance is actually a verbal auditory agnosia (i.e. inability to comprehend spoken language in the absence of hearing difficulties). Over time, affected children experience a gradual deterioration in verbal comprehension and production, eventually leading to failure to respond to non-verbal sounds and mutism. Seizure semiology varies, and 20-30% of patients do not exhibit clinical seizure activity at all. There is no clear relationship between the severity of the seizures and the severity of the language deficits. EEG shows predominantly bilateral posterior temporal spikes or spike-wave discharges that are activated by sleep. The clinical seizures are usually easy to control, with spontaneous resolution occurring in mid-adolescence. There are several treatment options, including corticosteroids, levetiracetam, benzodiazepines, and valproic acid. Language function improves when the active phase of spike-wave discharges resolves; however, if the aphasia begins in early childhood and/or persists for more than 1-2 years, long-term language deficits are almost universa

A 9-year-old girl has been having difficulty falling asleep at night for several months, and she is irritable and inattentive at school. She says that her legs tingle when she lies down to sleep and she has to keep moving them around to get the feeling to go away. Her mother has noticed that her daughter kicks her legs frequently when she sleeps. Otherwise she has been healthy and takes no medications. Her general and neurological examinations are normal. Which of the following laboratory studies should be obtained to further evaluate her condition? Nerve conduction velocity Somatosensory evoked potential Liver function tests Fasting glucose Ferritin

Correct Answer is: Ferritin This patient presents with features typical of restless legs syndrome, which is estimated to affect about 2% of children 8-17 years of age. It is characterized by an urge to move the legs, accompanied by unpleasant sensations. Symptoms are worse at night and relieved by movement. Primary RLS is idiopathic, while secondary RLS is caused by several conditions, including iron deficiency, pregnancy, renal disease, neuropathy, medications (including neuroleptics and serotonin reuptake inhibitors). Genetic factors play a prominent role. The condition adversely impacts ability to fall asleep, thereby causing insomnia. Most patients have periodic limb movements (stereotyped limb movements lasting 0.5-5 seconds recurring in 20-40 second intervals. When evaluating children with RLS, secondary causes should be sought, the most common of which is iron deficiency. Ferritin level below 50 warrants a trial of iron replacement. Dopamine agonists (ropinirole, pramipexole) are an effective treatment, as relative CNS dopamine deficiency seems to be the underlying etiology in primary RLS.

A 3-year-old child with a history of developmental delay is brought to your office for an evaluation. She is accompanied by her maternal grandmother, who has custody of her because of parent neglect. On physical exam, she has mild dysmorphic features with a head circumference at the 5th percentile and height/weight at 10th percentile. She has moderate to severe language delay. She has a smooth philtrum with a thin upper lip. Which of the following is the most likely diagnosis? Autism Down's syndrome Fetal alcohol syndrome Rett's syndrome Trisomy 18

Correct Answer is: Fetal alcohol syndrome The diagnosis of fetal alcohol syndrome is based on clinical history of maternal exposure to alcohol and the physical appearance of the child. The diagnostic criteria recommended by the Research Society of Alcoholism Fetal Alcohol study group require: a) presence of prenatal or postnatal growth retardation b) CNS dysfunction such as developmental delay or intellectual impairment; and c) at least two of the following: microcephaly, microphthalmia and short palpebral fissures, or all 3 and hypoplastic philtrum with thin upper lip

Paramedics are called to see a 5-month-old boy who vomited and then stopped breathing at home. The mother's boyfriend had been babysitting for the afternoon while the mother worked. He left the house abruptly when the mother returned. The infant had no prior history for significant health problems. However, his mother reported that he currently has an upper respiratory infection and has not been sleeping well. In the Emergency Department, the child is breathing regularly, but is unresponsive. A CT scan of the brain reveals a small amount of acute subdural bleeding in the interhemispheric fissure. Which of the following is the most appropriate next step in evaluation? Fundoscopic examination of the eyes Radiograph of the wrist for bone age determination Serum level of cortisol Serum copper and ceruloplasmin levels DNA mutation analysis for osteogenesis imperfecta

Correct Answer is: Fundoscopic examination of the eyes Abusive head trauma, also known by several other terms, including shaken baby syndrome and non-accidental trauma, is common, and accounts for a large proportion of traumatic brain injury (TBI) cases in children under 2 years of age. Typically, the history available at presentation does not match the pattern of physical injury. Symptoms are non-specific, including vomiting, apnea, seizures, and depressed consciousness. Subdural hemorrhage, commonly involving the interhemispheric fissure, is the single most common feature with brain imaging, but cerebral contusions, stroke, and hypoxic-ischemic injury may also occur. Associated problems may include retinal hemorrhage, injury to the cervical spine or abdomen, and fractures

A 17 year old boy is brought to your office by his mother because of episodes of loss of consciousness. He has had one of these episodes on 4 occasions over the past 6 months. His mother witnessed one of them. His mother says she was in another room and heard a loud thud. She ran into the room into which her son had just walked from outside and found him limp and unconscious on the floor. He remained unconscious with rapid, non-rhythmic eye flutter for 2-3 minutes, and then awakened but was confused and sleepy for almost an hour afterwards. He does not remember the episodes, but remembers dropping cups and feeling more clumsy prior to the episodes. The feeling of clumsiness occurs more upon awakening. There is no family history of epilepsy or seizures, and his past medical history is unremarkable. His general and neurological examinations are normal. His EEG is most likely to show: Generalized 6 Hz polyspike discharges on a slow background. No discharges on a normal background. Generalized 4-5 Hz spike-and-wave discharges on a normal background . Lateralized 6 Hz polyspike discharges on a normal background. Decremental beta discharges on a slow background.

Correct Answer is: Generalized 4-5 Hz spike-and-wave discharges on a normal background. This is an adolescent with juvenile myoclonic epilepsy, likely sometimes triggered by bright light (e.g., being outside in the sun). He is a cognitively normal adolescent and not likely to have background slowing on his interictal EEG. But discharges are common interictally in patients with this syndrome. Although it can run in families and at least one gene responsible for the syndrome is inherited as an autosomal dominant trait, it is not surprising that his family history is negative, as the inheritance patters of other associated genes and the genetics of the disorder in many patients are not understood.

An 8-year-old boy is referred for evaluation of a change in his gait, which began 2 years ago and has gradually worsened. His parents describe the problem as a tendency for his right foot to turn down and in. They noticed that he appears normal in the morning, but his foot begins to turn by late afternoon each day, even if he has not been physically active. His past medical history is otherwise unremarkable, and he is a good student in 3rd grade. He takes no medications. His father has similar but less severe symptoms which began in his early 20's. On examination, the boy is alert with intact cranial nerves. His strength is normal, but tone and tendon reflexes are increased in the distal right lower extremity. Plantar responses are flexor. Sensation is intact. Which of the following diagnostic studies is most likely to specify a diagnosis? ceruloplasmin genetic study for DYT1 Genetic studies for dopa-responsive dystonia MRI brain Genetic study for hereditary spastic paraplegia

Correct Answer is: Genetic studies for dopa-responsive dystonia Autosomal dominant GTP cyclohydrolase deficiency or Segawa disease is a dopa-responsive dystonia caused by a heterozygous mutation of the GCH1 gene on chromosome 14q22.1-2. It was previously known as hereditary progressive dystonia with marked diurnal fluctuation. The age of onset is usually around 6 years of age and starts with postural dystonia of a lower extremity, mostly with talipes equinovarus. Some patients develop action dystonia (especially retrocollis or oculogyric crisis). Postural high-frequency (8-10 Hz) tremor develops in an upper extremity at around 10 years of age. The symptoms are typically asymmetric and show marked diurnal fluctuation (aggravation towards the evening and near complete if not complete recovery after overnight sleep). Growth decelerates such that patients are 2 standard deviations below age norms for height by the late teens. Over time the dystonia becomes more generalized and tremor becomes more pronounced and become constant rather than fluctuating. There is a wide variation in age of onset and some families show anticipation. Brain CT and MRI scans are normal. CSF examination reveals low levels of homovanillic acid, biopterin, and neopterin. Treatment with levodopa/carbidopa 4-5 mg/kg/day alleviates the symptoms.

A 10-month-old boy has developed increasingly severe dystonia and rigidity since he became acutely ill 3 months ago. The illness began with fever and vomiting associated with hypoglycemia and metabolic acidosis for which he was hospitalized and treated with IV fluids. At that time he became acutely hypotonic with loss of head control and had seizures. An EEG was normal. MRI scan demonstrated atrophy of the frontoparietal areas with widening of the Sylvian fissures as well as increased T2 signal of the bilateral putamen and caudate nuclei. His past medical history is remarkable for large head size since early infancy. Family history is unremarkable. His examination is remarkable for severe diffuse rigidity, dystonia, and tongue thrusting. Which of the following is the most likely diagnosis? Glutaric acidemia type 1 MELAS Wilson's disease Carbon monoxide poisoning Methylmalonic aciduria

Correct Answer is: Glutaric acidemia type 1 Glutaric acidemia type 1 is an autosomal recessive inborn error of lysine, hydroxylisine, and tryptophan metabolism caused by deficiency of glutaryl-CoA dehydrogenase. This enzyme is located in the mitochondrial matrix. Over 60 mutations of the gene encoding this enzyme have been identified, including missense, nonsense, and intronic variants. There is a correlation between residual enzyme activity and biochemical phenotype, but not between phenotype and severity of the genetic lesion, as siblings within the same family and with the same mutation have discordant phenotypes. Nongenetic factors such as fever, infections, and fasting play an important role in precipitating neuronal damage. It is rare (worldwide incidence 1:100,000) except in certain populations such as the Old-Order Amish community of Pennsylvania and North American Ojibway-Cree in Canada. Patients have macrocephaly at birth or shortly thereafter, with atrophy of the frontoparietal cortical areas creating a "bat-wing" appearance (widening of the Sylvian fissures), creating a "micrencephalic macrocephaly". Patients are subject to subdural hemorrhages due to stretching of veins from large CSF extra-axial collections. Episodes of acute decompensation typically occur between 6 and 18 months and if not present by 5 years of age are unlikely to occur. These episodes are associated with edema of the putamen and caudate nuclei and present clinically with hypotonia and loss of head control. Seizure-like episodes are common, but EEG does not show epileptiform features. Hypotonia resolves over several weeks and is replaced by rigidity and dystonia. These patients are typically left with severe motor disability but most often have normal cognition. Diagnosis is usually made by urine organic acid analysis which shows elevated levels of 3-OH-glutaric acid. Urine glutarylcarnitine is also typically elevated (acylcarnitine urine testing). Identification of the condition on newborn screen allows for prevention of the acute decompensation episodes by use of a special diet with protein restriction, supplemented with lysine-free special formula and carnitine.

A 6-year-old white boy presents to the office for evaluation of developmental delay. He was born full-term following an unremarkable pregnancy but remained in the hospital for 2 days secondary to problems with feeding and jaundice. Birth weight was 7 lbs. 8 oz. (25-50%ile) and length was 22 in. (>95%ile). On examination, he is noted to be tall and macrocephalic (both > 97th percentile) with mildly dysmorphic facial features, including a high, broad forehead, down-slanting palpebral fissures, and a pointed chin. His hands and feet are enlarged. His neurologic examination is non-focal. There are no similarly affected individuals in the family. Which genetic test is most likely to yield a positive result? Chromosome microarray FISH for chromosome 22q11 deletion Karyotype Methylation analysis of chromosome 15q Nuclear receptor SET domain containing protein-1 (NSD1 testing)

Correct Answer is: Nuclear receptor SET domain containing protein-1 (NSD1 testing) Sotos syndrome, also known as cerebral gigantism, is a genetic condition that affects 1:10,000 to 1:50,000 individuals. The syndrome is characterized by prenatal and postnatal overgrowth, macrocephaly, and characteristic facial features—specifically a high anterior hair line; frontotemporal hair sparsity, a long, thin face; frontal bossing; downslanting palpebral fissures; and a prominent mandible. Advanced bone age is common but is not a universal finding. Neurologic hallmarks include hypotonia and global developmental delay, with most affected adults exhibiting mild intellectual disability. Sotos syndrome can be caused by either intragenic truncating mutations or microdeletions that result in haploinsufficiency of the nuclear receptor binding SET domain protein 1 (NSD1), located on chromosome 5q35.3. Intragenic mutations account for more than 80% of cases in European-origin white patients, wheres microdeletions occur with a higher incidence in the Japanese population. The majority of cases are sporadic, though autosomal dominant pedigrees have been report

An 18 month old girl who has been walking quite steadily for 4 or 5 months and has been speaking in easily understandable two-word phrases is suddenly seen to be unsteady on her feet and to wobble irregularly when sitting. Her speech is difficult to understand and she seems generally quite irritable. She is admitted to a local hospital for evaluation. A day after admission, she is noted to have irregular, jerky, conjugate movements of her eyes and is unable to bring a spoon smoothly to her mouth while eating. She may have a: Good prognosis medulloblastoma Bad prognosis medulloblastoma Good prognosis neuroblastoma Bad prognosis neuroblastoma Bad prognosis glioblastoma

Correct Answer is: Good prognosis neuroblastoma Opsoclonus-myoclonus-ataxia (OMA) syndrome is a paraneoplastic syndrome most often associated with neuroblastoma. The neuroblastomas with which it occurs tend to be the more differentiated, less malignant tumors, and often, the neuroblastoma can be cured, but the progressive course of OMA syndrome and its cognitive sequelae are the cause of longterm disability. Both anti-T-cell and anti-B-cell therapies are useful in this disorder.

An 8-year-old girl presents to your office with a 1-year history of abnormal movements including eye rolling, facial grimacing, and head jerking that occur multiple times per day. She describes a tingling sensation in her neck that is temporarily relieved by jerking her head. If she concentrates, she can stop the movements from happening for several seconds at a time. Her only medications are loratadine 10 mg daily, which was started two years ago to address her repetitive sniffing and throat clearing, and ibuprofen 400 mg as needed for neck pain related to her head jerking movements. Over the past year, she has stopped sniffing but has started audibly inhaling. She is doing well in 3rd grade, and neither her parents, nor her teachers have any concerns about her mood or behavior. Family history is pertinent for anxiety disorder in her mother and ADHD in her brother. Her general and neurologic examinations are normal, with the exception of the aforementioned adventitious movements. The most appropriate first-line agent to treat this patient's movements would be: Guanfacine Risperidone Pimozide Sertraline Haloperido

Correct Answer is: Guanfacine Tourette syndrome (TS) is a tic disorder, which affects an estimated 0.1 to 3% of the population. It is characterized by multiple motor and one or more vocal tics, which occur many times a day nearly every day for more than 1 year, though not necessarily concurrently. Onset must be prior to age 18 years, and the tics cannot be due to the effects of a substance or general medical condition. Many patients with TS have co-morbid obsessive-compulsive disorder (OCD), attention deficit hyperactivity disorder (ADHD), anxiety, depression, learning disorders, and/or oppositional behavior/conduct disorder. The decision to treat tics is based on whether the tics are causing morbidity. If so, pharmacotherapy should be initiated with low doses of -adrenergic drugs (clonidine, guanfacine). Although there is good empiric support for haloperidol, pimozide, and risperidone, these are usually reserved for patients who have failed an -adrenergic agent due to the fact that all three drugs are associated with significant side effects, including extrapyramidal symptoms, weight gain, and electrocardiographic changes

A 3-year-old boy is brought to the emergency room by his parents with a 3-day history of irritability and 2-day history of unsteadiness in walking and using his arms. He complains of pain in his legs and back and is now unable to walk without assistance. His past medical history is unremarkable except for a 1-week bout of gastroenteritis which resolved last week. He takes no medications. Family history is noncontributory. On examination, he has normal ocular movements and pupillary reactions. He has mild facial weakness. He has mild weakness in both shoulders, but a good handgrip. There is a coarse tremor when reaching for objects. He cannot support his own weight and is unable to sit unsupported. Deep tendon reflexes are absent and plantar responses are flexor. Sensation is intact. Which of the following is the most likely diagnosis? Acute cerebellar ataxia Acute transverse myelitis Botulism Guillain Barre syndrome Polymyositis

Correct Answer is: Guillain Barre syndrome The combination of facial and limb weakness with areflexia of acute onset and rapid progression are most compatable with acute inflammatory demyelinating polyradicuopathy, although a significant subset of these patients have an axonal form rather than demyelinating. Guillain Barre syndrome is an eponym for acute or chronic, axonal or demyelinating inflammatory neuropathy. Presentation can be different in young children than in adults. Up to one third of pediatric cases occur in children less than 3 years of age. In the pediatric group, pain is a common presenting complaint, involving the back or limbs. Ataxia and disturbances of equilibrium rarely observed in adults are present in up to 15% of children. Electrophysiologic studies performed in the first week of the illness most often demonstrate prolonged or absent F-responses (88%), prolonged distal latency (75%), conduction block (58%), and reduction in conduction velocity (50%).

A 13-year-old girl is admitted following a seizure, described as a tonic-clonic convulsion preceded by unresponsiveness with head and gaze deviation to the left side. Her behavior has also been increasingly peculiar over the past 2 days, with agitation and possible visual hallucinations. A similar episode, resolving over one week, had occurred 6 months previously, but no diagnosis had been identified. On examination she is afebrile, awake, but agitated, unable to settle in bed. Her gait is ataxic. Brain CT and MR imaging are normal. A lumbar puncture shows normal opening pressure and CSF profile. Her EEG shows generalized slowing with multifocal epileptiform discharges. Her complete blood count and serum chemistries are normal, as are serum thyroid levels. However, she has markedly elevated anti-thyroglobulin serum antibodies. Which of the following is the most likely diagnosis? Herpes simplex encephalitis Schizophrenia Idiopathic frontal lobe epilepsy Hashimoto encephalopathy Rasmussen chronic encephalitis

Correct Answer is: Hashimoto encephalopathy The pathogenesis of Hashimoto encephalopathy (HE) is unknown, but likely relates to the activity of autoantibodies against CNS antigens. Serum levels of thyroid hormone may be either normal or low for patients with this disorder. It is an important diagnosis to consider in children and adolescents with unexplained seizures, ataxia, myoclonus, encephalopathy, or atypical psychosis, since it can be successfully treated with corticosteroid administration. It is likely under-diagnosed in children and adolescents due to the failure to consider HE on the differential diagnosis. The diagnosis is usually made by identifying the presence of high serum levels of antithyroglobulin and anti-microsomal antibodies. A relapsing-remitting course may be observed.

A 15 year old girl is brought in by ambulance in a coma. Her mother, who is a single parent with a full-time job, says the girl has been overly weight and health conscious for almost a year and frequently uses her babysitting money to buy vitamins and other supplements at the local health food store. She worries that the girl eats these supplements instead of food. The intern in the emergency room noted that the girl was adequately oxygenated and had a normal heart rate, and so sent her to radiology for a stat MRI scan. The T1-weighted images showed increased signal in the striatum, thalamus, and internal capsule bilaterally. The T2-weighted images showed white matter abnormalities and MR spectroscopy demonstrated elevated glutamine and glutamate in the white matter. Her blood work demonstrates markedly elevated liver function tests and a serum ammonia of 360 mcg/dl. The supplements she has taken most likely include: Vitamin B12 Ginko biloba Selenium Kava kava Chinese red rice

Correct Answer is: Kava kava The clinical picture, high ammonia level, MRI scan findings, and MRS on this patient are suggestive of hepatic encephalopathy. Kava kava is an herbal derived from roots of the plant Piper methysticum, a member of the pepper family found in the Western and South Pacific. It has been proposed to be anxiolytic and used in patients with anxiety disorders and as treatment for insomnia, premenstrual syndrome and stress. Recently, concerns have arisen regarding the safety of kava products, in particular due to reports of liver injury. However, several groups have disputed the evidence for hepatotoxicity, suggesting that responsibility for liver injury lies with adulterants or concomitant drugs or herbals. Furthermore, the literature on liver injury from kava kava has included several incomplete or overlapping reports, and causality was rarely well shown. Nevertheless, there are a small number of cases of severe hepatic injury arising during therapy that are convincing.

At 5 years of age, a young boy received the diagnosis of attention deficit hyperactivity disorder, but was not prescribed medication treatment. By 7 years of age, he was having frequent twitching of his face and eye blinking behavior. By 8 years of age, he was disruptive in class by making frequent whistling noises. Which of the following is the most likely additional symptom that this patient may experience? Psychosis Dystonia Learning disorder Myoclonus Wing beating tremor

Correct Answer is: Learning disorder This patient demonstrates a classic history for Tourette's Disorder, which is defined as having frequent multiple motor tics and at least one verbal tic for 1 year in duration. The patient's initial symptoms can include ADD or ADHD, which often is evident prior to the onset of the tics. Anxiety, depression, obsessions, compulsions, and learning disabilities are common. Coprolalia or copropraxia are uncommon, but may occur. The presence of additional movement disorders, such as dystonia, a wing-beating tremor, or myoclonus should alert the clinician to additional potential diagnoses including myoclonus dystonia, Wilson's Disease, or neuroacanthocythosis

A 16-year-old boy presents to the emergency department with a 12-hour history of progressive weakness that started while he was seated in class. He describes feeling a stinging sensation in his arms that progressed to numbness and weakness to the level of his upper chest. He has not voided in several hours and is unable to walk without assistance. His upper extremities are moderately hypotonic and his lower extremities are flaccid. Formal strength testing reveals (R/L): biceps 4/4, triceps 3/3, wrist extension 4/4, with all other muscle groups tested being grade 0. He has a sensory level at T2, and deep tendon reflexes are absent throughout. MRI of the spine reveals an ill-defined intramedullary hyperintensity extending from C5 to T2 without significant cord expansion or enhancement. MRI of the head is unremarkable. Routine CSF studies are normal, and no oligoclonal bands are detected. At this time, which of the following would be the most appropriate treatment? Prednisone 40 mg p.o. daily with a slow taper IVIG 0.5 g/kg i.v. daily x 4 days Methylprednisolone 500 mg i.v. b.i.d. x 5 days Cyclophosphamide 800 mg/m2 i.v. every 4 weeks Interferon-β−1a 30 μg i.m. every week

Correct Answer is: Methylprednisolone 500 mg i.v. b.i.d. x 5 days Transverse myelitis (TM) is a heterogeneous disorder that can be idiopathic or can occur secondary to a wide array of inflammatory (post-infectious, demyelinating), metabolic, and vascular processes. Inflammatory demyelinating lesions, as seen in clinically isolated syndrome, multiple sclerosis, and neuromyelitis optica, are the most common cause of TM. Acute therapy for any of these demyelinating conditions consists of high-dose, intravenous corticosteroids for 3 to 5 days. A subsequent oral steroid taper over 4-6 weeks is recommended for patients with acute disseminated encephalomyelitis. Long-term therapy with immunosuppressants may decrease the relapse rate for patients with NMO. Non-demyelinating forms of transverse myelitis should be treated according to the underlying etiology, e.g. parenteral cyanocobalamin (vitamin B12) supplementation for subacute combined degeneration.

A 3-year-old girl began to have some trouble walking one week ago. Her condition progressed such that she is unable to bear weight and has trouble raising her arms above her head. She cannot crawl. There does not seem to be any pain at rest but with the Kernig and Brudzinski maneuvers there is discomfort in the hamstrings. Her reflexes are absent but her cranial nerves seem to be intact. Imaging of the brain performed in the ED is normal. Which of the following would be the most likely to confirm the diagnosis? CSF protein Repetitive nerve stimulation study MRI of the spine with contrast Nerve conduction velocity study Response to IVIG infusion

Correct Answer is: Nerve conduction velocity study The clinical description of acute demyelinating polyneuropathy is best documented by the nerve conduction studies. Typically, conduction velocity is slowed but there may be prominent delays in F waves and conduction blocks as well. CSF protein may not be elevated early in the clinical course and thus is not reliable for diagnostic confirmation. On occasion MRI may identify inflamed nerve roots but this is not a reliable way to make the diagnosis. Repetitive stimulation would be normal in these patients as there is no defect in neuromuscular transmission. Response to treatment is not a reliable approach to diagnosis, particularly a nonspecific therapy such as immunoglobulin

A 5 year old boy is brought to your office by his parents. They say he was completely normal until 3 years of age, when he began bumping into things and not responding to visual cues. His pediatrician referred him to a pediatric ophthalmologist who examined him and told them he had retinitis pigmentosa. He is now legally blind. A month ago his parents noticed that his behavior began to change. He would ask the same question over and over again and seemed not to be satisfied with the answer. In fact, he seemed to them to be anxious. In the past week, his teacher has questioned whether or not he understands what she is saying to him. His pediatrician had the boy's hearing tested and it is normal. You examine him and find him to perseverate on verbal and manual tasks, to be anxious, and to have occasional, single myoclonic jerks of one or the other of his arms or legs. His retinas have a tigroid appearance. You decide to send his blood for genetic testing. You are particularly interested in the gene that causes: Metachromatic Leukodystrophy Rett Syndrome Neuronal ceroid lipofuscinosis Morquio syndrome Zellweger syndrome

Correct Answer is: Neuronal ceroid lipofuscinosis Juvenile Batten disease is a neurodegenerative disorder that often has its onset in late toddlerhood or early childhood. It typically starts with pigmentary changes in the retina and progressive visual loss. This is followed by progressive dementia, myoclonus, and seizures. More than 60 different mutations in the CLN3 gene have been found associated with juvenile Batten disease. But the function of the CLN3 gene product is not completely understood. CLN3 protein is found in the membranes of many cell types; it appears to have some function related to lysosomal trafficking, pH, and size.

A 10-year-old boy experiences head to head impact with another player while playing soccer. The patient lay motionless on the ground, while the second boy walked away without difficulty. Referees and coaches found him able to open his eyes to shout, but unable to verbally respond for at least two minutes. He vomited on the field, but then was able to stand and walk to the sidelines with his father's assistance. Regarding this patient's injury, which one of the following is correct? Concussion occurs as a result of penetrating forces applied to the brain. Soccer is the sport most likely to result in concussion (mild traumatic brain injury) in the United States, as defined as an incidence rate for all children participating in the sport. This player should be evaluated by his coach, and sent home with his father. Neuropsychological tests one week after the incident may show a decline compared to preinjury baseline values. As a result of the first concussion, the risks associated with a second concussion are lower.

Correct Answer is: Neuropsychological tests one week after the incident may show a decline compared to preinjury baseline values. Concussion (or mild traumatic brain injury) associated with organized sports is common in children and teenagers. Ice hockey and American football appear to have the highest incidence rates. It has been established that athletes (both pediatric and adult) under-report the occurrence of symptoms resulting from concussion. Younger athletes may require more time to recover to baseline, and more conservative return-toplay decisions are appropriate for the pediatric age group. Children with one concussion are at higher risk for a second concussion. Neuropsychological testing may reveal deficits even in the absence of reported symptoms.

You are doing a consultation for a local school system on a particularly difficult student. He is an 8 year old boy who has always been "hard-to-handle". His teachers have always assumed that his parents are excessively lenient with him, particularly because they lost an older child to leukemia when this boy was 5 years old. His current teacher tells you he refuses to do anything she asks and argues indiscriminately with other children in the class. He is regularly disruptive, throwing balled-up paper around the room while the children are supposed to be working on a math or English problem. He seems quite bright to her, as he has an advanced vocabulary for age, including some words she would rather never hear in the classroom. This child should be evaluated for: Autistic spectrum disorder Attention deficit disorder Oppositional-defiant disorder Schizoaffective disorder Obsessive-compulsive disorder

Correct Answer is: Oppositional-defiant disorder Children with disruptive, antisocial behavior should be evaluated for oppositional-defiant disorder. This child does not have the sterotypies or lack of social interaction of a child with an autism spectrum disorder. He may seem to have difficulty paying attention or staying on task, but it seems this is because of the intrusions of his need to "act out" and oppose requests or needs of others. There is no evidence in the description of this youngster of a thought disorder or perseverative compulsions.

A 4-year-old girl experiences an event in which she is suddenly confused and pale. After approximately 10 minutes she began to vomit repeatedly, and upon her arrival in the Emergency Department she remained unresponsive, with tonic eye deviation to her right side. These symptoms resolved after the intravenous administration of lorazepam. Her brain MRI was normal, and EEG showed independent focal spikes over the occipital head regions bilaterally. In retrospect, she had a similar but slightly briefer event 6 months previously, which was attributed to food poisoning. Which of the following is the most likely diagnosis? Lennox-Gastaut syndrome Cyclic vomiting Panayiotopoulos syndrome Benign Rolandic epilepsy Landau-Kleffner syndrome

Correct Answer is: Panayiotopoulos syndrome Initially reported by Panayiotopoulos as a distinct clinical entity in 1989, this syndrome is now recognized by the International League Against Epilepsy (ILAE) as Early-Onset Benign Childhood Seizures with Occipital Spikes. It is an increasingly recognized syndrome, notable for mimicking other paroxysmal or acute disorders, such as syncope, gastroenteritis, or toxic ingestion. Autonomic symptoms such as pallor, mydriasis, and tachycardia are common. Individual seizures are usually at least 5 minutes in duration, and status epilepticus is common. However, seizures tend to be relatively infrequent, and remit for many affected patients.

A full-term newborn was being discharged on day 2 of life when his mother observed staring followed by chewing and then bilateral limb jerking. The nurse was called and witnessed the end of the episode in which the baby had rhythmic clonic activity of both upper extremities and face lasting approximately 30 seconds. It was estimated that the entire episode lasted 2 minutes. He had been born uneventfully with Apgar scores of 8/9. The infant's lab work immediately following the episode was unremarkable as was a cranial ultrasound scan. An interictal EEG was normal in active and quiet sleep with a normal state change. Family history revealed that the father was said to have had neonatal seizures which resolved by 3 months of age. Which of the following is the most likely underlying pathophysiological mechanism accounting for his condition? Unknown Acetylcholine receptor abnormality Sodium channel abnormality Potassium channel abnormality Glutamate metabolism abnormality

Correct Answer is: Potassium channel abnormality Benign familial neonatal convulsions (BFNC) is a well-recognized cause for neonatal seizures and the diagnosis is made by the family history of similar episodes in an infant with no risk factors for neonatal seizures and a normal interictal EEG. This syndrome has been estimated to occur in 14.4 per 100,000 live births. The pattern of inheritance is autosomal dominant with penetration of 85 percent. BFNC is one of several epileptic disorders characterized as a channelopathy. The two major genes identified in this condition are both voltage-gated potassium channels (KCNQ2, KCNQ3) that are responsible for the M-current. This current is a slowly repolarizing current that is responsible for modulating neuronal bursting by hyperpolarizing the cell membrane. In mice disruption of kcnq2 is lethal and only a 25% disruption of the M-current in humans is sufficient to produce BFNC (making the mutation of a single allele potentially pathologic and explaining the dominant mode of inheritance). Clinical features of BFNC are characterized by focal or multifocal clonic or tonic seizures, a family history of neonatal seizures, and no other neurologic abnormalities. The seizures typically occur within a few days to one week of life, although there is some variability in age at onset. Seizures are usually brief, but recurrence may occur until age two to three months, when spontaneous resolution typically occurs. The interictal EEG is usually normal. The treatment for benign familial neonatal convulsions is similar to that for other neonatal seizures

A 10-year-old boy presents to your office with difficulty running that has been gradually progressive over the past 2 years. He attributes this problem to intermittent low back pain, which is worsened by activity but does not radiate. He has also had intermittent urinary incontinence for the past 6 months. On examination, he has normal bulk but mildly increased tone in his distal lower extremities and mild weakness in ankle dorsiflexion bilaterally. Sensory examination is normal. Deep tendon reflexes are 2+ in the upper extremities and 3+ in the lower extremities. Plantar responses are flexor bilaterally. Rectal tone is normal. His spine is notable for lumbar levoscoliosis and a sacral hemangioma. He elevates his left shoulder and hip while walking but maintains a narrow base. Spinal magnetic resonance imaging shows a low-lying conus medullaris at L3 and an L4 hemivertebra with associated levoscoliosis. Surgical treatment is most likely to lead to improvement in the following symptoms except? Scoliosis Pain Spasticity Bladder function Weakness

Correct Answer is: Scoliosis Tethered cord syndrome (TCS) refers to a constellation of motor and sensory findings that are attributable to excessive tension on the spinal cord. The majority of cases are associated with spinal dysraphism; however, TCS can also occur when an intradural process (e.g. abnormal thickened, shortened filum terminale, lipoma of the filum, fibrous adhesions) interferes with normal ascension of the conus with respect to the vertebral column. TCS can present at any age. Infants may have cutaneous stigmata of spina bifida occulta, such as nevi, lipomas, tufts of hair, hemangiomas, and dermal sinuses. Other red flags in this age group include urinary dribbling, anorectal malformations, orthopedic deformities of the lower extremities, and scoliosis. Children and adolescents may be asymptomatic or may present with slowly progressive motor and sensory deficits. The clinical picture may also include varying degrees of weakness; patchy sensory deficits; scoliosis; nondermatomal pain in the low back, perineum, and/or legs; bladder or less frequently bowel incontinence; orthopedic deformities of the lower extremities; and gait disturbances. MRI is the study of choice in evaluating TCS, as it can detect both the level of the conus and the cause of the tethering. Symptomatic patients should be treated with surgical untethering, which results in pain relief in almost all cases and stabilization of neurologic decline in roughly 8090% of patients. Long term follow up in one cohort demonstrated scoliosis progression after surgical tethering. However there was improvement in muscle strength, gait, and spasticity. The role of surgery in asymptomatic patients remains less clear.

A 13 year old boy is morbidly obese. His pediatrician checks him annually for evidence of metabolic syndrome. This year, his fasting glucose was 160 and his fasting cholesterol was 240, so he was started on a sulfonylurea and a statin. Two weeks later, he awaked unable to lift either arm sufficiently to comb his hair. His proximal arm muscles ached as if he had done chin-ups in gym class. He is likely to have: Statin-induced myopathy Deconditioning syndrome FSH muscular dystrophy Endogenous steroid myopathy Hypothyroidism

Correct Answer is: Statin-induced myopathy An uncommon side effect of statins exists along a spectrum that includes myolysis, myopathy, and myalgias in various patients. It generally involves proximal musculature and is painful and reversible with early withdrawal of the statin.

A 2-month-old baby boy is brought to the physician by his parents because he is floppy compared to their first child. He does not lift his head or roll over and he perspires profusely on his head and face when feeding. He is alert and smiles appropriately but has scant limb movements and a weak cry. On examination the child appears hypotonic and weak. He is arreflexic and the muscles are difficult to palpate. A specific diagnosis is most likely to be established by which of the following studies? Serum CK Blood spot for alpha 1-4 glucosidase deficiency. Determination of CTG repeat number in the PK gene. Survival motor neuron gene mutation analysis Electromyography

Correct Answer is: Survival motor neuron gene mutation analysis Perspiration when feeding is an indication of the work the infant is doing to breath and eat at the same time. The expressive face would be expected in SMA at this stage but not in myotonic dystrophy (type 1 associated with PK gene abnormality). Although Pompe's disease would not affect the facial expression at this age, the fact that the muscles are not firm and easily palpated would be unexpected. Though the serum CK, EMG, MRI and muscle biopsy are reasonable tests to consider, the most common cause of hypotonia and weakness in this age group, particularly given the alert and expressive face and flabby muscles would SMA-1.

The parents of a 5 year old boy are quite distressed at his recent nocturnal episodes. He goes to sleep fine, but at around midnight two or three nights per week, he starts screaming and crying as if frightened. When they go into his room, they find him sitting upright with his eyes wide open, but they cannot reason with him or get him to calm down. It is as if he does not hear them at all. Eventually, he lies back down, seemingly returning to sleep. He is not incontinent during these episodes and does not have adventitious movements of his limbs or eyes. The next morning, he does not remember the episodes. His past medical history, family history, and physical examination are unremarkable. The best thing to tell his parents is that: The boy should have a video EEG as soon as possible. The boy is likely to outgrow these episodes. You are asking Child Protective Services to take the boy out of their custody. You would like the family to be seen by a psychiatrist. The boy needs to be treated with zolpidem.

Correct Answer is: The boy is likely to outgrow these episodes. Night terrors occur in young preschool and school age children. They are characterized by appearing to be awake but not being aware of surroundings or others in them. Screaming, fearfulness, excited babbling, and crying are characteristic as well. They are very frightening for the observers, but the child is not aware they occurred the next morning. They typically remit before age 10 years. Reassurance of the family is the only thing usually needed.

A previously healthy 16 year old girl comes to your office complaining of 6 months of episodic vertigo, with each episode lasting hours to days. There is no headache associated with the episodes, but she has "always" had occasional bifrontal, pulsatile headaches without photo- or phonophobia or vertigo. These occasional headaches respond well to ibuprofen. Her mother tells you no one in the family gets migraines, but she gets sinus headaches unaccompanied by fever or nasal drainage that require her to lie down in a quiet, dark room. The girl's younger brother gets nauseated if he tries to read in the back seat of the car. You examine her and fine two 0.5-cm café au lait spots, one on the right thigh and one on the left flank. An MRI scan her pediatrician obtained prior to her visit with you showed a 3 mm Chiari I malformation without abnormalities of the ventricles, brain parenchyma, or extra-axial spaces. You should explain to this girl and her family the relationship of her vertigo to: The anatomic abnormality of her brain seen on MRI scan. Her mother's chronic sinusitis. The family's predisposition to migraine and its equivalents. Her café au lait spots and likely neurofibromatosis. A brain tumor too small to be seen on the MRI scan

Correct Answer is: The family's predisposition to migraine and its equivalents. Benign paroxysmal vertigo is a "migraine equivalent" that segregates with migraine in families. This girl's mother's headaches sound more like migraines than like sinus headaches, and her brother's motion sickness is also a migraine equivalent. Her café au lait spots are neither large nor numerous enough to be of consequence. Her Chiari I malformation is also likely a variant of no medical significance

A 4 month old infant is seen by her pediatrician for constipation. The pediatrician puts her on Karo syrup. Three days later, her mother brings her back to the pediatrician's office because she is not feeding well. Her mother also thinks her eyelids look droopy and her muscle tone is floppier than usual. She appears listless and hypotonic to the pediatrician, who sends her to the emergency department of a local hospital. The emergency room physician asks the mother if anything new or different is being fed to the baby or is present in the baby's environment. The mother says she has not fed the baby anything other than breast milk, but she and her husband are having an extension put on their home for which they just broke ground 3 weeks ago. There is no family history of neuromuscular disease and no one else has been sick at home. The baby is hypotonic, areflexic, and has ptosis and dysconjugate gaze although she seems to be awake and alert. An EMG demonstrates an incremental response to rapid repetitive nerve stimulation. The baby's condition is likely the result of: The Karo syrup being given before 2 years of age The recent digging up of soil outside of the home An occult viral infection A genetic disorder of muscle A congenital disorder of the neuromuscular junction

Correct Answer is: The recent digging up of soil outside of the home Infant botulism often starts with constipation and rapidly progresses to include areflexic hypotonia and paresis of eye movement. It can be the result of ingestion of Clostridium botulinum spores in honey; more often, it is the result of spores in soil that is unearthed during construction. EMG findings include an incremental response to rapid repetitive nerve stimulation.

A 17-year-old girl who has a 4 year history of intermittent migraines treated with sumatriptan nasal spray, now presents to her neurologist with a new type of pain. She describes a constant, burning pain with numbness on the right side of her face. There is no fever or other sign of illness. She recently had four wisdom teeth extracted. Her past medical history is otherwise unremarkable and her family history is pertinent for migraines in her mother and grandmother. Her examination demonstrates subjective decreased sensation to touch over the right cheek. Which of the following is the most likely diagnosis? Glossopharyngeal neuralgia Nervus intermedius neuralgia Periodic migrainous neuralgia Trigeminal dysesthesia Trigeminal neuralgia

Correct Answer is: Trigeminal dysesthesia Trigeminal dysesthesia is defined as a continuous pain following complete or partial damage to a peripheral nerve. Wisdom tooth extraction or root canal is a frequent cause. The discomfort can be self-limiting, depending on nerve regeneration. Treatment can be with local anesthetic blockade, topical application of capsaicin, or oral tricyclic antidepressants. Periodic migrainous neuralgia is a type of cluster headache that presents as orofacial pain, usually in the maxilla, and is not precipitated by oral surgery, but is a primary headache disorder. Trigeminal neuralgia is a painful, unilateral occurrence of brief, electric, shock-like pain limited to one or more divisions of the trigeminal nerve. The pain is often evoked by minor stimuli, such as washing the face or brushing the teeth. It is unrelated to oral surgery, and is usually due to a tumor, such as acoustic neuroma, or a demyelinating lesion. Glossopharyngeal neuralgia similar in quality to that of trigeminal neuralgia, but occurs in the distribution of the glossopharyngeal nerve, often triggererd by chewing, swallowing, or rotating the head. It is due to an elongated stylohyoid process that irritates or compresses the nerve, and radiograph shows a calcified stylohyoid ligament; treatment is resection of this ligament with microvascular decompression of the nerve. Nervus intermedius neuralgia is localized to the middle ear, otherwise described similarly to trigeminal neuralgia.

The family of a 7 year old girl decided not to have her get any immunizations because her 10 year old brother has autism they believe was caused by his MMR shot. Two months ago, she had a febrile illness characterized by an itchy rash and upper respiratory symptoms. That illness remitted uneventfully. Yesterday, she and her family noticed the sudden onset of weakness of her right arm and drooping of the right side of her mouth with slurred speech. Today, these are unchanged. An MRI of her brain shows a small infarct of the anterior limb of her left internal capsule. An MRA shows occlusion of lenticulostriate branch of the middle cerebral artery. This affliction could probably have been avoided if the girl had been given a vaccine against: Varicella Measles, mumps, and rubella Diphtheria, pertussis, and tetanus Hemophilus influenzae type b Swine flu

Correct Answer is: Varicella Ischemic infarction is a known sequela of varicella in childhood. Some estimate that it at one time accounted for as much as 1/3 of strokes seen in young children. However, the routine vaccination of preschool-age children against varicella has greatly reduced the incidence of varicella and its complications and sequelae. This disease and its sequelae are, however, still seen in children in populations and regions in which immunization is not the rule

A 4 month old boy is electively admitted to the hospital for evaluation of failure to thrive, developmental delay, and generalized seizures. He is found to have leukopenia. His evaluation includes urine organic acids and he is also found to have methylmalonic aciduria. He might have an abnormality involving utilization of: Vitamin D Vitamin E Vitamin C Vitamin B12 Vitamin A

Correct Answer is: Vitamin B12 Vitamin B12-responsive methylmalonic acidemia (MA) is an inborn error of vitamin B12 (cobalamin) metabolism characterized by recurrent ketoacidotic comas or transient vomiting, dehydration, hypotonia and intellectual deficit, which responds to vitamin B12. There are three types: cblA, cblB and cblD-variant 2 (cblDv2). To date, over 120 patients with cblA, 66 patients with cblB and 6 patients with cblDv2 have been reported. Prevalence of 1/48,000-1/61,000 have been reported for MA of all causes in North America, and 1/26,000 in China, but only a subset of this is vitamin B12-responsive MA. Patients usually present in infancy or early childhood with features including lethargy, failure to thrive, recurrent vomiting, dehydration, respiratory distress, muscle hypotonia, hepatomegaly and coma. They may also show signs of anemia (not megaloblastic), have potentially life-threatening ketoacidosis and/or hyperammonemia, and developmental delay and intellectual deficit, with metabolic stroke affecting the brain stem. MA frequently leads to end-stage renal failure by adolescence or adulthood. Patients with cblB are usually more severely affected than patients with cblA

A 17-year-old Caucasian female presented to her physician's office with a 3-4 month history of blurry vision, headache and difficulty ambulating. Her examination is remarkable for bilateral papilledema, multiple retinal angiomas, dysmetria, ataxia, brisk deep tendon reflexes, extensor plantar responses, and ankle clonus. An urgent brain MRI scan reveals a large enhancing midline cerebellar lesion compressing the fourth ventricle and brainstem. A cervical MRI scan reveals multiple nodular enhancing lesions. Which of the following is the most likely diagnosis? Medulloblastoma Cerebellar astrocytoma Arteriovenous malformation Von Hippel Lindau disease Ependymoma

Correct Answer is: Von Hippel Lindau disease In patients with a family history of von Hippel Lindau disease, diagnosis can be made by the presence of a single retinal angioma or cerebellar hemangioblastoma, renal cell carcinoma, or pheochromocytoma. Without such a family history, the diagnosis is made on the basis of 2 or more retinal angiomas, hemangioblastomas within the central nervous system, or one such lesion and a characteristic visceral tumor. Hemangioblastomas can affect not only the retina and cerebellum, but the brainstem and spinal cord as well, thus resulting in heterogeneous neurological findings. Many factors influence the prognosis of patients with von Hippel Lindau disease, including brain and spinal cord localization, the underlying biology of the central nervous system tumors, and comorbid entities such as carcinomas. Because of the potential for multi-organ involvement, a multi-disciplinary team of specialists is required to properly manage patients with von Hippel Lindau disease.

A full-term baby boy was born after an uneventful labor but appeared depressed at birth with generalized hypotonia and hypoventilation. He was transferred to the NICU for observation and noted to have episodes of clonic seizure activity involving left and right side independently lasting 30-60 seconds on several occasions. His exam showed dysmorphic features consisting of micrognathia, broad nasal bridge, high forehead, dolichocephalic skull with large anterior and posterior fonatanelles. A firm liver edge was noted 1 cm below the costal margin. His neurological exam showed marked generalized hypotonia with poor gag and suck/swallow, absent reflexes and difficult to elicit Moro response. The EEG showed multifocal spikes on a mildly low voltage background without state change but no electrographic seizures. CT scan showed dilated posterior horns (colopocephaly). Which of the following is the most likely diagnosis? Zellweger's syndrome Prader-Willi syndrome Myotonic dystrophy Spinal muscular atrophy Congenital CMV infection

Correct Answer is: Zellweger's syndrome Zellweger's syndrome, an autosomal recessive condition, is the severest form of the peroxisomal biogenesis disorders (PBD) and presents at birth with dysmorphic features, liver cirrhosis, severe hypotonia and neonatal encephalopathy with seizures. The diagnosis is suggested by the dysmorphic features and confirmed by analysis of very long chain fatty acids (elevated VLCFA C26:0, C26:1; abnormal ratios of C24/C22; C26/C22). Peroxisomes (which are missing in Zellweger's) process VLCFA which, if they accumulate abnormally, affect CNS and other cell membranes. CNS dysmyelination occurs and in the severe form leads to problems with radial glial guides and migrational disorders (pachygyria, polymicrogyria, neuronal heterotopias, colopocephaly) resulting in severe encephalopathy and seizures. Mutations in twelve different PEX genes, those that encode peroxins, the proteins required for normal peroxisome assembly, have been identified in Zellweger's syndrome. Treatment is symptomatic and children fail to develop, with death usually in the first year. Other PBD are neonatal adrenoleukodystrophy, infantile Refsum disease (both considered part of the Zellweger's Syndrome Spectrum) and rhizomelic chondrodysplasia punctata. These disorders can be separated clinically and 80% of PBD are among the Zellweger's Syndrome Spectrum with an incidence of 1 in 50,000 births in the US

A previously healthy 8-year-old girl presents to the emergency room with vomiting and lethargy after a holiday party. Her past medical history is unremarkable. Early developmental milestones were normal and she is an average student in the 2nd grade. Family history is remarkable for a maternal cousin who died in early infancy from an unspecified condition. On examination she is obtunded. EEG shows slowing and triphasic waves. Which of the following is the most appropriate next step in evaluation? ammonia CSF exam plasma lactate MRI of the brain with MRS CT brain scan

Correct Answer is: ammonia Although classically associated with presentation in infancy, the majority of individuals with urea cycle disorders (UCDs) present outside the neonatal period, frequently in childhood. Signs and symptoms are often vague, but recurrent; fulminant presentations associated with acute illness are also common. A disorder of urea cycle metabolism should be considered in children who have recurrent symptoms, especially neurologic abnormalities associated with periods of decompensation. Routine laboratory tests, including measurement of plasma ammonia concentrations, can indicate a potential UCD; however, specific metabolic testing and ultimately enzymatic or molecular confirmation are necessary to establish a diagnosis. Treatment with dietary protein restriction and medications may be challenging in children.

A 4-year-old boy is referred for evaluation of behavior changes. His mother reports he was always a little delayed in his development, but over the past two months he has become more subdued and less interested in playing with his siblings. Use of language has also decreased; he is now speaking in two word phrases whereas he had been using full sentences. No other focal abnormalities are present on neurologic examination. He has no prior medical illnesses. His mother reports that she was recently evaluated by her primary care doctor for recurrent pulmonary infections and had a positive HIV test. Examination is notable for bilateral mild spastic paraparesis. The child's ELISA and Western blot studies confirm he is infected with human immunodeficiency virus. His brain MRI shows global cerebral atrophy and bilaterally symmetric hyperintense signal in the white matter on T2-weighted sequences; these same regions appear isointense on T1-weighted images. Spinal fluid examination reveals 10 WBCs/hpf, protein 40 mg/dL, glucose 65 mg/dL. India ink and AFB stains are negative. PCR and serology of the spinal fluid for CMV is negative. Toxoplasmosis serology (IgM and IgG) are negative. Serum and CSF cryptococcal antigen testing is normal. PCR of the spinal fluid for JC virus DNA is sent and pending. Which of the following treatments is most likely to provide significant clinical improvement in this child? amphotericin anti-retroviral therapy gangcyclovir isoniazid and rifampin pyrimethamine and sulfadiazine

Correct Answer is: anti-retroviral therapy This patient shows signs and symptoms of progressive encephalopathy and spastic diparesis in the setting of HIV infection. Given his mother's history, this was most likely acquired prenatally. The differential diagnosis in this setting includes several different infectious agents, as well as primary CNS lymphoma. HIV encephalopathy is a likely possibility. Toxoplasmosis is unlikely given the normal serology and lack of mass lesions on imaging. Tuberculous infection usually produces a striking basilar meningitis. Although a negative acid-fast stain of the CSF does not exclude this possibility, the child's clinical and radiology presentation is not consistent with CNS tuberculosis. The laboratory markers for both cryptococcal and CMV meningitis were negative. The two most likely entities resulting in the child's condition are progressive multifocal leukoencephalopathy (PML) and HIV encephalopathy. Although both can present with a subacute encephalopthy, PML is unlikely to produce isolated cognitive decline, without other focal neurologic deficits (especially visual field loss). Neuroimaging of PML also shows more focal abnormalities, extending into the subcortical white matter, and tend to be hypointense on T1-weighted images. The incidence of congenital HIV encephalopathy has decreased greatly in developed countries since the use of highly active anti-retroviral therapy (HAART) in affected women. There are multiple presentations for this disorder, including a progressive, static or indolent course. Pre-existing global developmental delay is not unusual in the progressive course. After a varied period of time, the child usually develops a combination of behavioral deterioration, cognitive regression and motor impairment (with both pyramidal and extra-pyramidal findings possible on neurologic examination). Diagnosis is one of exclusion, with neuroimaging and spinal fluid examination necessary to rule out the many other infectious and neoplastic processes contributing to encephalopathy in children with HIV infection. The CSF in HIV encephalopathy is often normal, though can have a mildly increased cell count and protein level. The most common radiographic findings include cortical atrophy, non-specific white matter changes and basal ganglia calcifications (best seen on computed tomography.) Aggressive use of anti-retroviral therapy has been shown to either slow or reverse the features of progressive HIV encephalopathy in both pediatric and adult patients

A 6-month-old infant was normal at birth, but has shown developmental regression with progressive opisthotonus, stridor, and swallowing difficulties. His examination also demonstrates enlarged spleen, squint, head retroflexion, spasticity, and trismus. Analysis of which of the following tissues would most likely define his condition? bone marrow blood leukocytes muscle skin live

Correct Answer is: blood leukocytes The diagnosis of Gaucher disease relies on demonstration of deficient glucosylceramidase enzyme activity in peripheral blood leukocytes. Carrier testing by assay of enzyme activity is unreliable because of overlap in enzyme activity between carriers and non-carriers. Identification of two disease-causing alleles in GBA, the only gene known to be associated with Gaucher disease, provides additional confirmation of the diagnosis

A 7-month-old boy is referred due to periods of irritability, stiffness and regression of skills since 4 months of age. He had an unremarkable birth and early development. He also has periods of elevated temperature without evidence of infection. Examination shows no dysmorphic features. There are episodes of opisthotonus in response to tactile stimulus. Muscle tone is increased and reflexes are absent. A spinal tap showed elevated protein but no other abnormalities. Which of the following laboratory studies would most likely be abnormal in this patient? Phenylalanine hydroxylase 7-dehydrocholesterol acid maltase galactocerebrosidase beta hexosamindase A

Correct Answer is: galactocerebrosidase Krabbe disease is characterized by an infantile-onset progressive neurologic regressive disorder with death before age two years of age in the majority of individuals. Alternatively onset may be between age six months and the fifth decade with slower disease progression. Children with the infantile form appear to be normal for the first few months of life but show extreme irritability, spasticity, and developmental delay before age six months; psychomotor regression progresses to a decerebrate state with no voluntary movement. In the late-onset forms, individuals can be clinically normal until weakness, vision loss, and intellectual regression become evident. The clinical course is variable even within the same family. Stage I is characterized by irritability, stiffness, arrest of motor and mental development, and episodes of temperature elevation without infection, possibly caused by involvement of the hypothalamus. The child, apparently normal for the first few months after birth, becomes hypersensitive to auditory, tactile, or visual stimuli and begins to cry frequently without apparent cause. In almost all individuals with Krabbe disease, galactocerebrosidase (GALC) enzyme activity is deficienct in leukocytes isolated from whole heparinized blood or in cultured skin fibroblasts.

A 5-year-old boy is brought to the emergency room after having 2 episodes of clonic seizures involving his right arm and leg, each of which lasted 10 minutes. He has no chronic medical illnesses. He and several family members were ill with a gastrointestinal illness several weeks ago. On examination in the emergency room, his blood pressure is 140/94, he is afebrile and pulse is 140/min. Neurologic exam is notable for weakness and decreased tone of his right arm greater than leg and an extensor plantar response on the right side. Pertinent laboratory studies reveal a hemoglobin of 7.6, hematocrit of 25 and platelet count of 25,000. The peripheral smear shows multiple schistocytes. Serum sodium is 134, bicarbonate 20, glucose 165, BUN 40, creatinine 3.9. A non-contrast head CT scan is unremarkable. Which of the following is the most likely diagnosis for this patient? hemolytic-uremic syndrome hypertensive encephalopathy hyponatremia Kawasaki syndrome sickle cell vasculopathy

Correct Answer is: hemolytic-uremic syndrome The above patient has symptomatic focal seizures secondary to a recent ischemic stroke. Hyponatremia is a cause of symptomatic seizures, though not secondary to stroke. Typically, serum sodium level need to be less than 120-125 meq/L to produce seizures. The remainder of the choices listed above are potential causes of seizures and ischemic stroke, though hypertensive encephalopathy usually produces bilaterally symmetric abnormalities on imaging, especially in the more posterior brain regions. Focal infarctions respecting vascular distributions are not commonly seen. The triad of hemolytic anemia, acute renal failure and thrombocytopenia is most suggestive of hemolytic-uremic syndrome. Although sickle cell disease can cause anemia with seizures secondary to ischemic stroke, sickle cells and not schistocytes (red blood cells that have been fragmented by passage through damaged small blood vessels) would be noted on the peripheral smear. Hemolytic-uremic syndrome is most commonly triggered by a diarrheal associated illness, often shiga-toxin producing strains of E. coli. The toxins have a direct effect on the kidney, causing injury to the subendothelial capillary spaces, leading to acute renal failure. Several other infectious, immunologic, familial and endocrine causes have been reported. Neurologic manifestations are seen in 30-40% of cases, and are most commonly a combination of behavioral changes, motor seizures, stroke and varying degrees of encephalopathy. Treatment at this time remains largely supportive, with renal dialysis required in 30-50% of cases. Plasmapheresis has been utilized in some cases

A 7-year-old boy is brought to the physician by his mother, who wants to have him started on methylphenidate for ADHD treatment. He has a history of speech delay, impaired social functioning with his school peers, restricted interests with a fascination for dinosaurs, and hand flapping behaviors when he is excited. In addition, his parents and teachers report that he has a very short attention span, poor impulse control, and hyperactive behavior. His older brother has been diagnosed with ADHD and has shown a dramatic benefit from methylphenidate treatment, with minimal adverse effects. He does not, however, have the same language and social difficulties as this patient. In considering treatment of this patient with methylphenidate, which of the following statements most accurately reflects the likelihood of positive and negative medication effects compared with his brother's response? equally effective with equal side effects less effective and greater side effects less effective but fewer side effects more effective but greater side effects more effective with fewer side effects

Correct Answer is: less effective and greater side effects Autism spectrum disorder (ASD) is characterized by impairments in social functioning and communication, and restricted, repetitive behaviors and interests. ADHD is characterized by difficulties in maintaining attention and patterns of impulsive and/or hyperactive behavior. The DSM-IV-TR precludes a comorbid diagnosis of ASD and ADHD, arguing that because ASD is the more severe disorder within the diagnostic hierarchy, any inattention, hyperactivity, or impulsivity observed can be thought of as stemming from the primary diagnosis of autism. This exclusion fails to account for the many individuals with ASD who also meet full criteria for ADHD; these individuals typically have more severe overall impairment. Studies have shown that patients with both ASD and ADHD have a lower likelihood of favorable response with stimulant treatment (25-50%) when compared to patients with isolated ADHD; the incidence of adverse effects in this population is up to 60%, mainly agitation

A 7-year-old boy presents for evaluation of developmental delay. He has history of frequent ear infections and a heart murmur due to valvular disease. His examination demonstrates short stature, lordotic posture, slightly coarse facial features, and corneal clouding. Which of the following laboratory tests would most likely define his condition? amino acid analysis 7-dehydrocholesterol level lysosomal enzyme screen CSF neurotransmitter levels uric acid level

Correct Answer is: lysosomal enzyme screen The mucopolysaccharidoses (MPSs) are a group of rare inherited lysosomal storage disorders caused by the deficiency or absence of specific lysosomal enzymes. The absence of these enzymes leads to accumulation of complex carbohydrates in cells and tissues and in the cellular organelles, the lysosomes. These complex carbohydrates are also known as mucopolysaccharides or glycosaminoglycans (GAGs) serve as the building blocks for connective tissues. MPS III, or Sanfilippo syndrome, is the most common of MPS disorders and results from the deficiency or absence of 4 different enzymes that are necessary to degrade the GAG heparan sulfate. Each enzyme deficiency defines a different form of Sanfilippo syndrome. Individuals with type A lack the enzyme heparan sulfate sulfatase. Individuals with type B lack the enzyme N -acetyl-alpha-D-glucosaminidase. Patients with type C lack acetyl-CoA:alpha-glucosaminide acetyltransferase. Patients with type D lack the enzyme Nacetylglucosamine-6-sulfatase. The particular form of Sanfilippo syndrome cannot be determined based on clinical features, but rather must rely on enzymatic assays. The neurological features encompass mental regression beginning in the toddler years, coarse facial appearance, but may be mild, sleep disturbance and progressive myelopathy due to storage material in the spine. Seizures may occur. Initial clinical signs present as a change in behavior. Patients are hyperactive and display aggressive and destructive behaviors.

A 16-month-old boy is evaluated for microcephaly and developmental delay. He and his family migrated to the United States recently from a village in Bosnia. His mother had 2 other children (with different fathers) who both have mild intellectual disability, microcephaly, and delay in linguistic development, congenital heart defects and severe growth retardation. The mother reports eating a "vegetarian" diet as a child but currently does not practice any dietary restrictions. She barely graduated high school and has difficulty remembering appointments and paying bills. Which of the following is the most likely explanation for this child's condition? chromosome disorder infantile autism Leigh's disease maternal phenylketonuria urea cycle disorder

Correct Answer is: maternal phenylketonuria Phenylketonuria (PKU) is an autosomal recessive disorder of amino acid metabolism affecting approximately 1/5000-10,000 infants in North America. It is most often due to deficiency of the enzyme phenylalanine hydroxylase which causes the accumulation of harmful metabolites, including phenylketones. If untreated, PKU leads to mental retardation, seizures, psychoses, eczema and a distinctive "mousy" odor. Pregnant women who have PKU and are not maintaining dietary restrictions are at risk of delivering infants with the effects of untreated maternal PKU and may also be at risk of developing neurologic symptoms themselves if they don't maintain lifelong dietary restrictions. The characteristic features of maternal PKU syndrome include mental retardation, microcephaly, intrauterine growth retardation, and congenital heart defects. The risk for these defects is increased with maternal plasma phenylalanine levels >20 mg/dL).

A 3-year-old boy is admitted to the pediatric ward with fever for one week and progressive vomiting and complaints of headache. He has no current respiratory symptoms, though his mother has been sick with intermittent respiratory symptoms for the past several months. On the day after admission, he complains of double vision and his speech becomes dysarthric. CT scan of the brain is performed and shows enlarged ventricles without any obvious mass lesion. Neurologic examination reveals bilateral papilledema, an incomplete bilateral abducens paresis and mild right facial weakness. He is also noted to have some difficulties with swallowing liquids. A lumbar puncture is performed and shows 250 WBC's (lymphocyte predominant), protein of 350 mg/dL and glucose of 30. Opening pressure was 300 mm. Routine gram stain and bacterial culture of the cerebrospinal fluid are negative at 48 hours. MRI scan of the brain reveals enhancement of the basilar meninges and two nodular enhancing lesions in the temporal lobes. Which of the following investigations would be most appropriate to order on the remaining spinal fluid? enterovirus PCR herpes simplex virus PCR human herpesvirus PCR lymphocytic choriomeningitis virus PCR mycobacterium tuberculosis PCR

Correct Answer is: mycobacterium tuberculosis PCR The above patient has neurologic and radiographic findings consistent with a basilar meningitis. The cranial nerve palsies typically develop from a thickened exudate at the base of the brain that either entraps the cranial nerve itself or leads to a vasculitis and ischemia of the nerve. This exudative process suggests a more localized inflammatory process, seen more commonly with tubercular, Lyme, fungal and parasitic meningitis. This process is also seen in cases of sarcoidosis, neoplastic meningitis and some forms of CNS vasculitis. It is extremely unusual for a viral meningitis to cause a basilar meningitis presentation. Lymphocytic choriomeningitis virus causes and intrauterine infection with clinical and radiologic features that mimic congenital CMV infection and would not present in the above fashion. Central nervous system tuberculosis follows the dissemination of tuberculosis after primary infection (most commonly the lung). This process can occur at the same time as the primary disease or several years later. Multiplying organisms in the subarachnoid space cause a basilar meningitis and at times parenchymal infections called tuberculomas (noted on this patient's MRI scan.) Hydrocephalus is also seen frequently. Tuberculosis can also involve the spinal cord, either from a vertebral body abscess or an arachnoiditis. Diagnosis of central nervous system tuberculosis can be quite challenging. A positive acid-fast stain is seen in only 5-25% of patients. Isolation of the bacteria by routine single lumbar puncture (AFB culture) is noted in 20-40% of patients. Increasing the volume collected to 10 mL (20 mL or more in adult patients) and performing repeated sampling of the spinal fluid yield a positive result in up to 50-80%. PCR assay has a sensitivity on a single sample of 5075%. Treatment of CNS tuberculosis is quite challenging. Initial therapy involves four drugs (combination of isoniazid, rifampin, pyrazinamide and ethambutol or streptomycin) for 2 months. As sensitivity tests allow, the treatment can usually be pared down to two agents. Response to treatment is usually seen within 2 weeks, though therapy should continue for 9-12 months. Treatment of the primary TB infection reduces the incidence of CNS infection. A shunting procedure of the CSF is occasionally required and corticosteroids may reduce the mortality and morbidity of the disease, especially in children with altered level of consciousness, papilledema, focal deficits or elevated intracranial pressure.

A 9-year-old boy is admitted for sudden headache, leading rapidly to depressed level of consciousness and weakness on the left side of the body. He had been watching television with his family. There was no recent history for fever, trauma, or systemic complaints. He was previously well and took no medications. On examination he was arousable with deep pain, but with gaze preference to his left and lack of motor response to the left arm or leg. CT imaging of the brain in the Emergency Department revealed a large intraparenchymal hemorrhage within the right parieto-occipital region with evidence of herniation of brain tissue from right to left under the falx. An emergency craniotomy was performed to evacuate the hemorrhage, and subsequent angiography demonstrated an arteriovenous malformation (AVM) in the right parietal midline region fed predominately by the right posterior cerebral artery. Which of the following symptoms would be most likely have been present in this child prior to rupture of the AVM? episodes of left sided weakness learning disability migraine with aura no symptoms prior to the hemorrhage simple partial sensory seizures

Correct Answer is: no symptoms prior to the hemorrhage After the newborn period, the most common cause of non-traumatic intraparenchymal brain hemorrhage in children is an AVM. For 75-80% of children and adolescents, there are no symptoms prior to the hemorrhage itself. Relative to bleeding from AVMs in adults, there is a higher likelihood of hemorrhage into the posterior fossa or deep cerebral structures, and the bleeding itself is often more massive. The morbidity and mortality with hemorrhage from AVMs in children is high, and a proactive approach to treatment is usually recommended when they are discovered. Surgical treatment should be individualized to the circumstances of each case, but can include craniotomy with surgical excision, endovascular embolization, or stereotactic radiosurgery (or some combination of these).

A 5-day-old baby boy has had weakness of the right arm since birth. He was born by vaginal delivery which was complicated by shoulder dystocia. His birthweight was 4500 gm. His mother had gestational diabetes mellitus. Immediately after birth his entire right upper extremity was weak and areflexic, but over time he has improved. Now, he has normal grasp and finger extension, but is unable to flex at the elbow or abduct at the shoulder and his biceps reflex is absent. In addition to range of motion exercises, which of the following is the best management recommendation at this time? apply a splint to the elbow to maintain flexion perform MRI of the cervical spine perform nerve conduction velocity study observation only surgical exploration for nerve grafting

Correct Answer is: observation only Obstetrical brachial plexus palsy is defined as a flaccid paresis of an upper extremity due to traumatic stretching of the brachial plexus received at birth. The incidence ranges from 0.38-3 per 1000 live births. The most significant risk factor is high birth weight, and occurs 3 times more frequently when the birth weight is >4500 gm. Other risk factors include breech delivery and maternal factors include diabetes mellitus, obesity, excessive weight gain, age >35 years, maternal pelvic anatomy, and primiparity. Shoulder dystocia is a strong predictor for brachial plexus injury. Associated injuries include clavicle and humerous fractures, facial nerve injury, cephalhematoma, and torticollis. Injuries include neurapraxia, neuroma, rupture, and avulsion. Upper plexus injury (C5 and C6) is also called Erb's palsy and is the most common type. In these patients, the adducted and internally rotated at the shoulder, extended at the elbow, flexed at the wrist, and extended at the fingers (the waiter's tip posture). Total plexus injury is second most common type and is more likely to result in long term disability. The majority (70-95%) of patients make complete spontaneous recovery. Evaluation using electromyography, nerve conduction studies, somatosensory evoked potentials, and MRI of the cervical spine can be utilized if recovery at 3 months is incomplete, although the optimal timing of these studies is controversial. The initial goal of management is to maintain passive range of motion, supple joints, and muscle strength. Indications for surgical exploration and reconstruction of the brachial plexus include failure of recovery of elbow flexion and shoulder abduction from the 3rd to the 6th month of life. It may take up to 2 years for complete recovery.

A 5-day-old baby boy has had weakness of the right arm since birth. He was born by vaginal delivery which was complicated by shoulder dystocia. His birthweight was 4500 gm. His mother had gestational diabetes mellitus. Immediately after birth his entire right upper extremity was weak and areflexic, but over time he has improved. Now, he has normal grasp and finger extension, but is unable to flex at the elbow or abduct at the shoulder and his biceps reflex is absent. In addition to range of motion exercises, which of the following is the best management recommendation at this time? apply a splint to the elbow to maintain flexion perform MRI of the cervical spine perform nerve conduction velocity study observation only surgical exploration for nerve grafting

Correct Answer is: observation only Obstetrical brachial plexus palsy is defined as a flaccid paresis of an upper extremity due to traumatic stretching of the brachial plexus received at birth. The incidence ranges from 0.38-3 per 1000 live births. The most significant risk factor is high birth weight, and occurs 3 times more frequently when the birth weight is >4500 gm. Other risk factors include breech delivery and maternal factors include diabetes mellitus, obesity, excessive weight gain, age >35 years, maternal pelvic anatomy, and primiparity. Shoulder dystocia is a strong predictor for brachial plexus injury. Associated injuries include clavicle and humerous fractures, facial nerve injury, cephalhematoma, and torticollis. Injuries include neurapraxia, neuroma, rupture, and avulsion. Upper plexus injury (C5 and C6) is also called Erb's palsy and is the most common type. In these patients, the adducted and internally rotated at the shoulder, extended at the elbow, flexed at the wrist, and extended at the fingers (the waiter's tip posture). Total plexus injury is second most common type and is more likely to result in long term disability. The majority (70-95%) of patients make complete spontaneous recovery. Evaluation using electromyography, nerve conduction studies, somatosensory evoked potentials, and MRI of the cervical spine can be utilized if recovery at 3 months is incomplete, although the optimal timing of these studies is controversial. The initial goal of management is to maintain passive range of motion, supple joints, and muscle strength. Indications for surgical exploration and reconstruction of the brachial plexus include failure of recovery of elbow flexion and shoulder abduction from the 3rd to the 6th month of life. It may take up to 2 years for complete recovery.

A 10-year-old boy is admitted with new onset status epilepticus. He was seen by his pediatrician 3 days prior to admission for a large, tender lymph node in the right neck. A 10 day course of Augmentin was prescribed, and he was told to follow up in one week. On the morning of admission his mother had difficulty getting him to arouse and brought him to the ER. In the ER he developed right face and arm clonic activity, which stopped after he was given 3 doses of lorazepam and a loading dose of fosphenytoin. Thereafter, an EEG showed intermittent non-convulsive electrographic seizures from the left hemisphere. CT scan of the brain was normal. A lumbar puncture revealed a white blood count of 18 cells/mm3, with normal glucose and protein levels. The patient was started on vancomycin, ceftriaxone and acyclovir. Herpes simplex PCR testing was negative. Which of the following laboratory tests is most likely to be abnormal? acid fast bacilli culture of the cerebrospinal fluid cerebrospinal fluid for cryptococcal antigen repeat cerebrospinal fluid PCR for HSV I/II serum Bartonella henselae titers serum ELISA for Borrelia burgdorferi

Correct Answer is: serum Bartonella henselae titers Bartonella infections are most often acquired by a recent inoculation through a skin abrasion or scratch from a young kitten. Classically, affected patients demonstrate prominent lymphadenopathy in the cervical, axillary or inguinal region. Other systemic symptoms such as fever, anorexia, fatigue and headache may also be present. Atypical cases of cat scratch disease do occur, often with neurologic features. Encephalopathy (presenting with mental status changes or new-onset seizures, often status epilepticus) is the most common neurologic presentation, representing about 2% of cases of cat scratch disease in the largest case series reviewed. Less likely neurologic presentations include acute or subacute monocular blindness due to retinitis, a peripheral neuritis resembling Guillain-Barre syndrome, meningitis and a transverse myelitis. Fever is not necessarily present during the encephalopathy and cerebrospinal fluid exam may not show pleocytosis. Hyperintense lesions can be seen on T2-weighted and FLAIR MRI in the deep grey structures and rarely the cortex, though imaging studies are typically normal. A high degree of suspicion for the disease should be maintained in anyone with acute or subacute encephalopathy and either lymphadenopathy and/or recent kitten exposure. Although multiple antibiotic regimens have been utilized in patients with focal brain lesions or encephalitis from bartonella infections, their benefit is uncertain. Spontaneous resolution is the rule for this disease, though rarely chronic epilepsy may develop. Lyme disease can present with a number of neurologic features, though seizures and status epilepticus are not common. Subacute or chronic meningitis and unilateral or bilateral facial nerve palsies are the most common neurologic presentations in children. Rarely, initial PCR testing for herpes simplex encephalitis can have a false negative result. In cases with a high index of suspicion, especially with focal seizures and hemorrhagic infarctions in the temporal or orbital frontal cortices, repeat PCR testing should be considered. Both cryptococcal and M. tuberculosis infections of the central nervous system would typically present in an immune compromised host. Although atypical mycobacterial infections can lead to prominent cervical lymphadenopathy, this feature is not present in M. tuberculosis infection


Conjuntos de estudio relacionados

APUSH Historical Evidence, Periods 1-9

View Set

Lesson 2: Citric Acid Cycle (Krebs Cycle) & The Electron Transport Chain

View Set

Chapter 8: Supporting Your Ideas

View Set

CIST 1220 SQL Final Exam Study Guide

View Set

HDFS 186 Consumer Economics Exam 2

View Set

Chapter 10: Moral Hazard, Systemic Risks, and Bailout

View Set